Anda di halaman 1dari 192

A 56 year old man is undergoing a radical nephrectomy via a posterior approach.

Which of the following structures is most likely to be encountered during the


operative approach?
A. 8th rib
B. 10th rib
C. 6th rib
D. 12th rib
E. 9th rib
The 11th and 12th ribs lie posterior to the kidneys and may be encountered during a
posterior approach. A pneumothorax is a recognised complication of this type of
surgery.
Renal anatomy
Each kidney is about 11cm long, 5cm wide and 3cm thick. They are located in a deep
gutter alongside the projecting verterbral bodies, on the anterior surface of psoas
major. In most cases the left kidney lies approximately 1.5cm higher than the right.
The upper pole of both kidneys approximates with the 11th rib (beware pneumothorax
during nephrectomy). On the left hand side the hilum is located at the L1 vertebral
level and the right kidney at level L1-2. The lower border of the kidneys is usually
alongside L3.
The table below shows the anatomical relations of the kidneys:
Relations
Relations Right Kidney
Posterior Quadratus lumborum, diaphragm,
psoas major, transversus abdominis
Anterior Hepatic flexure of colon
Superior Liver, adrenal gland

Left Kidney
Quadratus lumborum, diaphragm,
psoas major, transversus abdominis
Stomach, Pancreatic tail
Spleen, adrenal gland

Fascial covering
Each kidney and suprarenal gland is enclosed within a common and layer of investing
fascia that is derived from the transversalis fascia into anterior and posterior layers
(Gerotas fascia).
Renal structure
Kidneys are surrounded by an outer cortex and an inner medulla which usually
contains between 6 and 10 pyramidal structures. The papilla marks the innermost apex
of these. They terminate at the renal pelvis, into the ureter.
Lying in a hollow within the kidney is the renal sinus. This contains:
1. Branches of the renal artery
2. Tributaries of the renal vein

3. Major and minor calyces's


4. Fat
Structures at the renal hilum
The renal vein lies most anteriorly, then renal artery (it is an end artery) and the ureter
lies most posterior.
A 44 year old lady is undergoing an abdominal hysterectomy and the ureter is
identified during the ligation of the uterine artery. At which site does it insert into the
bladder?
A. Posterior
B. Apex
C. Anterior
D. Base
E. Superior aspect of the lateral side
Theme from September 2012 Exam
The ureters enter the bladder at the upper lateral aspect of the base of the bladder.
They are about 5cm apart from each other in the empty bladder. Internally this aspect
is contained within the bladder trigone.
Ureter

25-35 cm long

Muscular tube lined by transitional epithelium

Surrounded by thick muscular coat. Becomes 3 muscular layers as it crosses


the bony pelvis

Retroperitoneal structure overlying transverse processes L2-L5

Lies anterior to bifurcation of iliac vessels

Blood supply is segmental; renal artery, aortic branches, gonadal branches,


common iliac and internal iliac

Lies beneath the uterine artery

A 6 month old child is brought to the surgical clinic because of non descended testes.
What is the main structure that determines the descent path of the testicle?

A. Processus vaginalis
B. Cremaster
C. Mesorchium
D. Inguinal canal
E. Gubernaculum
Theme from April 2012 Exam
The gubernaculum is a ridge of mesenchymal tissue that connects the testis to the
inferior aspect of the scrotum. Early in embryonic development the gubernaculum is
long and the testis are located on the posterior abdominal wall. During foetal growth
the body grows relative to the gubernaculum, with resultant descent of the testis.
Testicular embryology
Until the end of foetal life the testicles are located within the abdominal cavity. They
are initially located on the posterior abdominal wall on a level with the upper lumbar
vertebrae (L2). Attached to the inferior aspect of the testis is the gubernaculum testis
which extends caudally to the inguinal region, through the canal and down to the
superficial skin. Both the testis and the gubernaculum are extra-peritoneal.
As the foetus grows the gubernaculum becomes progressively shorter. It carries the
peritoneum of the anterior abdominal wall (the processus vaginalis). As the processus
vaginalis descends the testis is guided by the gubernaculum down the posterior
abdominal wall and the back of the processus vaginalis into the scrotum.
By the third month of foetal life the testes are located in the iliac fossae, by the
seventh they lie at the level of the deep inguinal ring.
The processus vaginalis usually closes after birth, but may persist and be the site of
indirect hernias. Part closure may result in development of cysts on the cord.
A 28 year old man requires a urethral catheter to be inserted prior to undergoing a
splenectomy. Where is the first site of resistance to be encountered on inserting the
catheter?
A. Bulbar urethra
B. Membranous urethra
C. Internal sphincter
D. Prostatic urethra
E. Bladder neck
Theme from 2011 exam
Theme from January 2012 Exam

The membranous urethra is the least distensible portion of the urethra. This is due to
the fact that it is surrounded by the external sphincter.
Urethral anatomy
Female urethra
The female urethra is shorter and more acutely angulated than the male urethra. It is
an extra-peritoneal structure and embedded in the endopelvic fascia. The neck of the
bladder is subjected to transmitted intra-abdominal pressure and therefore deficiency
in this area may result in stress urinary incontinence. Between the layers of the
urogenital diaphragm the female urethra is surrounded by the external urethral
sphincter, this is innervated by the pudendal nerve. It ultimately lies anterior to the
vaginal orifice.
Male urethra
In males the urethra is much longer and is divided into four parts.
Pre-prostatic
urethra

Extremely short and lies between the bladder and prostate gland.It
has a stellate lumen and is between 1 and 1.5cm long.Innervated by
sympathetic noradrenergic fibres, as this region is composed of
striated muscles bundles they may contract and prevent retrograde
ejaculation.
Prostatic
This segment is wider than the membranous urethra and contains
urethra
several openings for the transmission of semen (at the midpoint of
the urethral crest).
Membranous Narrowest part of the urethra and surrounded by external sphincter. It
urethra
traverses the perineal membrane 2.5cm postero-inferior to the
symphysis pubis.
Penile urethra Travels through the corpus songiosum on the underside of the penis.
It is the longest urethral segment.It is dilated at its origin as the
infrabulbar fossa and again in the gland penis as the navicular fossa.
The bulbo-urethral glands open into the spongiose section of the
urethra 2.5cm below the perineal membrane.
The urothelium is transitional in nature near to the bladder and becomes squamous
more distally.
A 23 year old man undergoes an orchidectomy. The right testicular vein is ligated;
into which structure does it drain?
A. Right renal vein
B. Inferior vena cava
C. Common iliac vein
D. Internal iliac vein
E. External iliac vein

Theme from April 2012 Exam


The testicular venous drainage begins in the septa and these veins together with those
of the tunica vasculosa converge on the posterior border of the testis as the
pampiniform plexus. The pampiniform plexus drains to the testicular vein. The left
testicular vein drains into the left renal vein. The right testicular vein drains into the
inferior vena cava.
Scrotal and testicular anatomy
Spermatic cord
Formed by the vas deferens and is covered by the following structures:
Layer
Origin
Internal spermatic fascia
Transversalis fascia
Cremasteric fascia
From the fascial coverings of internal oblique
External spermatic fascia
External oblique aponeurosis
Contents of the cord
Vas deferens
Testicular artery
Artery of vas deferens
Cremasteric artery
Pampiniform plexus
Sympathetic nerve fibres
Genital branch of the
genitofemoral nerve
Lymphatic vessels

Transmits sperm and accessory gland


secretions
Branch of abdominal aorta supplies testis and
epididymis
Arises from inferior vesical artery
Arises from inferior epigastic artery
Venous plexus, drains into right or left
testicular vein
Lie on arteries, the parasympathetic fibres lie
on the vas
Supplies cremaster
Drain to lumbar and para-aortic nodes

Scrotum

Composed of skin and closely attached dartos fascia.

Arterial supply from the anterior and posterior scrotal arteries

Lymphatic drainage to the inguinal lymph nodes

Parietal layer of the tunica vaginalis is the innermost layer

Testes

The testes are surrounded by the tunica vaginalis (closed peritoneal sac). The
parietal layer of the tunica vaginalis adjacent to the internal spermatic fascia.

The testicular arteries arise from the aorta immediately inferiorly to the renal
arteries.

The pampiniform plexus drains into the testicular veins, the left drains into the
left renal vein and the right into the inferior vena cava.

Lymphatic drainage is to the para-aortic nodes.

A 21 year old man has an inguinal hernia and is undergoing a surgical repair. As the
surgeons approach the inguinal canal they expose the superficial inguinal ring. Which
of the following forms the lateral edge of this structure?
A. Inferior epigastric artery
B. Conjoint tendon
C. Rectus abdominis muscle
D. External oblique aponeurosis
E. Transversalis fascia
The external oblique aponeurosis forms the anterior wall of the inguinal canal and
also the lateral edge of the superficial inguinal ring. The rectus abdominis lies
posteromedially and the transversalis posterior to this.
Inguinal canal
Location

Above the inguinal ligament

The inguinal canal is 4cm long

Boundaries of the inguinal canal


Floor
External oblique aponeurosis

Inguinal ligament

Lacunar ligament

Internal oblique

Transversus abdominis

Roof

Anterior wall
Posterior wall

External oblique aponeurosis

Transversalis fascia

Conjoint tendon

Internal ring

Fibres of internal oblique

External ring

Conjoint tendon

Laterally

Medially

Contents
Males Spermatic cord and
ilioinguinal nerve

As it passes through the canal the spermatic


cord has 3 coverings:

External spermatic fascia from external


oblique aponeurosis

Cremasteric fascia

Internal spermatic fascia

Females Round ligament of uterus


and ilioinguinal nerve
Related anatomy of the inguinal region
The boundaries of Hesselbachs triangle are commonly tested and illustrated below:

Image sourced from Wikipedia

The image below demonstrates the close relationship of the vessels to the lower limb
with the inguinal canal. A fact to be borne in mind when repairing hernial defects in
this region.

Image sourced from Wikipedia

A 67 year old man is undergoing a transurethral resection of a bladder tumour using


diathermy. Suddenly during the procedure the patients leg begins to twitch.
Stimulation of which of the following nerves is the most likely cause?

A. Femoral
B. Pudendal
C. Sciatic
D. Obturator
E. Gluteal
Theme from January 2011 Exam
The obturator nerve is most closely related to the bladder (see below)
Page 209 brs ana

Image sourced from Wikipedia

Obturator nerve
The obturator nerve arises from L2, L3 and L4 by branches from the ventral divisions
of each of these nerve roots. L3 forms the main contribution and the second lumbar
branch is occasionally absent. These branches unite in the substance of psoas major,
descending vertically in its posterior part to emerge from its medial border at the
lateral margin of the sacrum. It then crosses the sacroiliac joint to enter the lesser
pelvis, it descends on obturator internus to enter the obturator groove. In the lesser
pelvis the nerve lies lateral to the internal iliac vessels and ureter, and is joined by the
obturator vessels lateral to the ovary or ductus deferens.

Supplies

Medial compartment of thigh

Muscles supplied: external obturator, adductor longus, adductor brevis,


adductor magnus (not the lower part-sciatic nerve), gracilis

The cutaneous branch is often absent. When present, it passes between gracilis
and adductor longus near the middle part of the thigh, and supplies the skin
and fascia of the distal two thirds of the medial aspect.

Obturator canal

Connects the pelvis and thigh: contains the obturator artery, vein, nerve which
divides into anterior and posterior branches.

Cadaveric cross section demonstrating relationships of the obturator nerve


A 45 year old lady is
undergoing a
Whipples procedure
for carcinoma of the
pancreatic head. The
bile duct is
transected. Which of
the following
vessels is mainly
responsible for the
blood supply to the
bile duct?
A. Cystic artery
Image sourced
Wikipedia
B.from
Hepatic
artery

C. Portal vein
D. Left gastric artery
E. None of the above
The bile duct has an axial blood supply which is derived from the hepatic artery and
from retroduodenal branches of the gastroduodenal artery. Unlike the liver there is no
contribution by the portal vein to the blood supply of the bile duct. Damage to the
hepatic artery during a difficult cholecystectomy is a recognised cause of bile duct
strictures.

Gallbladder

Fibromuscular sac with capacity of 50ml

Columnar epithelium

Relations of the gallbladder


Anterior
Liver
Posterior
Covered by peritoneum

Laterally
Medially

Transverse colon

1st part of the duodenum

Right lobe of liver


Quadrate lobe of liver

Arterial supply
Cystic artery (branch of Right hepatic artery)
Venous drainage
Cystic vein
Nerve supply
Sympathetic- mid thoracic spinal cord, Parasympathetic- anterior vagal trunk
Common bile duct
Origin
Relations at
origin

Relations
distally

Confluence of cystic and common hepatic ducts

Medially - Hepatic artery

Posteriorly- Portal vein

Duodenum - anteriorly

Pancreas - medially and laterally

Right renal vein - posteriorly

Arterial supply Branches of hepatic artery and retroduodenal branches of


gastroduodenal artery
Calot's triangle

Medially
Common hepatic duct
Inferiorly
Cystic duct
Superiorly
Inferior edge of liver
Contents
Cystic artery
7 year old boy presents with right iliac fossa pain and there is a clinical suspicion that
appendicitis is present. From which of the following embryological structures is the
appendix derived?
A. Vitello-intestinal duct
B. Uranchus
C. Foregut
D. Hindgut
E. Midgut
The appendix is derived from the midgut
It is derived from the midgut which is why early appendicitis may present with
periumbilical pain.
Appendix

Location: Base of caecum.

Up to 10cm long.

Mainly lymphoid tissue (Hence mesenteric adenitis may mimic appendicitis).

Caecal taenia coli converge at base of appendix and form a longitudinal


muscle cover over the appendix. This convergence should facilitate its
identification at surgery if it is retrocaecal and difficult to find (which it can be
when people start doing appendicectomies!)

Arterial supply: Appendicular artery (branch of the ileocolic).

It is intra peritoneal.

McBurney's point

1/3 of the way along a line drawn from the Anterior Superior Iliac Spine to the
Umbilicus

6 Positions:

Retrocaecal 74%

Pelvic 21%

Postileal

Subcaecal

Paracaecal

Preileal

A 34 year old man presents to the surgical clinic 8 months following a laparotomy for
a ruptured spleen. He complains of a nodule in the centre of his laparotomy wound.
This is explored surgically and a stitch granuloma is found and excised. From which
of the following cell types do granulomata arise?
A. Polymorpho nucleocytes
B. Plasma cells
C. Reed- Sternberg cells
D. Platelets
E. Macrophages
Granulomas are organised collections of macrophages
Theme from 2011 Exam
Macrophages give origin to granulomas.
Chronic inflammation
Overview
Chronic inflammation may occur secondary to acute inflammation.In most cases
chronic inflammation occurs as a primary process. These may be broadly viewed as
being one of three main processes:

Persisting infection with certain organisms such as Mycobacterium


tuberculosis which results in delayed type hypersensitivity reactions and
inflammation.

Prolonged exposure to non-biodegradable substances such as silica or suture


materials which may induce an inflammatory response.

Autoimmune conditions involving antibodies formed against host antigens.

Acute vs. Chronic inflammation


Acute inflammation
Changes to existing vascular structure and
increased permeability of endothelial cells
Infiltration of neutrophils
Process may resolve with:

Suppuration

Complete resolution

Abscess formation

Progression to chronic inflammation

Healing by fibrosis

Chronic inflammation
Angiogenesis predominates
Macrophages, plasma cells and
lymphocytes predominate
Healing by fibrosis is the main
result

Granulomatous inflammation
A granuloma consists of a microscopic aggregation of macrophages (with epithelial
type arrangement =epitheliod). Large giant cells may be found at the periphery of
granulomas.
Mediators
Growth factors released by activated macrophages include agents such as interferon
and fibroblast growth factor (plus many more). Some of these such as interferons may
have systemic features resulting in systemic symptoms and signs, which may be
present in individuals with long standing chronic inflammation.
The finding of granulomas is pathognomonic of chronic inflammation, as illustrated
in this biopsy from a patient with colonic Crohns disease

Image sourced from Wikipedia

A 53 year old man is undergoing a radical gastrectomy for carcinoma of the stomach.
Which of the following structures will need to be divided to gain access to the coeliac
axis?
A. Lesser omentum
B. Greater omentum
C. Falciform ligament
D. Median arcuate ligament
E. Gastrosplenic ligament
The lesser omentum will need to be divided. During a radical gastrectomy this forms
one of the nodal stations that will need to be taken.
Coeliac axis
The coeliac axis has three main branches.

Left gastric

Hepatic: branches-Right Gastric, Gastroduodenal, Right Gastroepiploic,


Superior Pancreaticoduodenal, Cystic.

Splenic: branches- Pancreatic, Short Gastric, Left Gastroepiploic

It occasionally gives off one of the inferior phrenic arteries.

Image sourced from Wikipedia

Relations
Anteriorly
Lesser omentum
Right
Right coeliac ganglion and caudate process of liver
Left
Left coeliac ganglion and gastric cardia
Inferiorly
Upper border of pancreas and renal vein
A 17 year old lady presents with right iliac fossa pain and diagnosed as having acute
appendicitis. You take her to theatre to perform a laparoscopic appendicectomy.
During the procedure the scrub nurse distracts you and you inadvertently avulse the
appendicular artery. The ensuing haemorrhage is likely to be supplied directly from
which of the following vessels?
A. Inferior mesenteric artery
B. Superior mesenteric artery
C. Ileo-colic artery
D. Internal iliac artery
E. None of the above
The appendicular artery is a branch of the ileocolic artery.
Appendix

Location: Base of caecum.

Up to 10cm long.

Mainly lymphoid tissue (Hence mesenteric adenitis may mimic appendicitis).

Caecal taenia coli converge at base of appendix and form a longitudinal


muscle cover over the appendix. This convergence should facilitate its
identification at surgery if it is retrocaecal and difficult to find (which it can be
when people start doing appendicectomies!)

Arterial supply: Appendicular artery (branch of the ileocolic).

It is intra peritoneal.

McBurney's point

1/3 of the way along a line drawn from the Anterior Superior Iliac Spine to the
Umbilicus

6 Positions:

Retrocaecal 74%

Pelvic 21%

Postileal

Subcaecal

Paracaecal

Preileal

A 63 year old man who smokes heavily presents with dyspepsia. He is tested
and found to be positive for helicobacter pylori infection. One evening he has
an episode of haematemesis and collapses. What is the most likely vessel to be
responsible?
A. Portal vein
B. Short gastric arteries
C. Superior mesenteric artery
D. Gastroduodenal artery
E. None of the above

Theme from January 2012 exam

He is most likely to have a posteriorly sited duodenal ulcer. These can invade
the gastroduodenal artery and present with major bleeding. Although gastric
ulcers may invade vessels they do not tend to produce major bleeding of this
nature.
Gastroduodenal artery
Supplies
Pylorus, proximal part of the duodenum, and indirectly to the pancreatic head
(via the anterior and posterior superior pancreaticoduodenal arteries)
Path
Most commonly arises from the common hepatic artery of the coeliac trunk
Terminates by bifurcating into the right gastroepiploic artery and the superior
pancreaticoduodenal artery
Image showing stomach reflected superiorly to illustrate the relationship of
the gastroduodenal artery to the first part of the duodenum

Image sourced

Which of the following nerves is responsible for the cremasteric reflex?


A. Lateral femoral cutaneous nerve
B. Femoral nerve
C. Obturator nerve

D. Genitofemoral nerve
E. None of the above
The motor and sensory fibres of the genitofemoral nerve are tested in the cremasteric
reflex. A small contribution is also played by the ilioinguinal nerve and thus the reflex
may be lost following an inguinal hernia repair.
Genitofemoral nerve
Supplies
- Small area of the upper medial thigh

pg 162 brs

Path
- Arises from the first and second lumbar nerves
- Passes obliquely through Psoas major, and emerges from its medial border opposite
the fibrocartilage between the third and fourth lumbar vertebrae.
- It then descends on the surface of Psoas major, under cover of the peritoneum
- Divides into genital and femoral branches.
- The genital branch passes through the inguinal canal, within the spermatic cord, to
supply the skin overlying the skin and fascia of the scrotum. The femoral branch
enters the thigh posterior to the inguinal ligament, lateral to the femoral artery. It
supplies an area of skin and fascia over the femoral triangle.

It may be injured during abdominal or pelvic surgery, or during inguinal hernia


repairs.

1
2
Rate question: 3
4
5

A 73 year old lady is admitted with brisk rectal bleeding. Despite attempts at
resuscitation the bleeding proceeds to cause haemodynamic compromise. An upper GI
endoscopy is normal. A mesenteric angiogram is performed and a contrast blush is
seen in the region of the sigmoid colon. The radiologist decides to embolise the vessel
supplying this area. At what spinal level does it leave the aorta?
A. L2
B. L1
C. L4

D. L3
E. T10
Theme from 2009 Exam
The inferior mesenteric artery leaves the aorta at L3. It supplies the left colon and
sigmoid. It's proximal continuation to communicate with the middle colic artery is via
the marginal artery.
Levels
Transpyloric plane
Level of the body of L1

Pylorus stomach

Left kidney hilum (L1- left one!)

Right hilum of the kidney (1.5cm lower than the left)

Fundus of the gallbladder

Neck of pancreas

Duodenojejunal flexure

Superior mesenteric artery

Portal vein

Left and right colic flexure

Root of the transverse mesocolon

2nd part of the duodenum

Upper part of conus medullaris

Spleen

Can be identified by asking the supine patient to sit up without using their arms. The
plane is located where the lateral border of the rectus muscle crosses the costal
margin.
Anatomical planes

Subcostal plane
Intercrestal plane
Intertubercular plane

Lowest margin of 10th costal cartilage


Level of body L4 (highest point of iliac crest)
Level of body L5

Renal arteries
COELIC AXIS
Left colon into sigmoid
Inferior mesenteric artery
Superior mesenteric artery

L2
L1
L2-L3
L3
L1

Bifurcation of aorta into common iliac arteries L4


Formation of IVC
L5 (union of common iliac veins)
Diaphragm apertures
Vena cava T8

Oesophagus T10

Aortic hiatus T12

43 year old man is undergoing a right hemicolectomy and the ileo-colic artery is
ligated. From which of the following vessels is is derived?
A. Inferior mesenteric artery
B. Superior mesenteric artery
C. Coeliac axis
D. Aorta
E. None of the above
The ileocolic artery is a branch of the SMA and supplies the right colon and terminal
ileum. The transverse colon is supplied by the middle colic artery. As veins
accompany arteries in the mesentery and are lined by lymphatics, high ligation is the
norm in cancer resections. The ileo-colic artery branches off the SMA near the
duodenum.
Colon anatomy
The colon is about 1.5m long although this can vary considerably.
Components:

Ascending colon

Transverse colon

Descending colon

Sigmoid colon

Arterial supply
Superior mesenteric artery and inferior mesenteric artery: linked by the marginal
artery.
Ascending colon: ileocolic and right colic arteries
Transverse colon: middle colic artery
Descending and sigmoid colon: left colic artery
Venous drainage
From regional veins (that accompany arteries) to superior and inferior mesenteric vein
Lymphatic drainage
Initially along nodal chains that accompany supplying arteries, then para-aortic nodes.
Embryology
Midgut- Second part of duodenum to 2/3 transverse colon
Hindgut- Distal 1/3 transverse colon to anus
Peritoneal location
The right and left colon are part intraperitoneal and part extraperitoneal. The sigmoid
and transverse colon are generally wholly intraperitoneal. This has implications for
the sequelae of perforations, which will tend to result in generalised peritonitis in the
wholly intra peritoneal segments.
A 53 year old man is undergoing a distal pancreatectomy for trauma. Which of the
following vessels is responsible for the arterial supply to the tail of the pancreas?
A.

Splenic artery

B.

Pancreaticoduodenal artery

C.

Gastric artery

D.

Hepatic artery

E.

Superior mesenteric artery

Pancreatic head is supplied by the pancreaticoduodenal artery


Pancreatic tail is supplied by BRS 326 branches of the splenic artery
There is an arterial "watershed" in the supply between the head and tail of the
pancreas. The head is supplied by the pancreaticoduodenal artery and the tail is
supplied by branches of the splenic artery.
Pancreas
The pancreas is a retroperitoneal organ and lies posterior to the stomach. It may be

accessed surgically by dividing the peritoneal reflection that connects the greater
omentum to the transverse colon. The pancreatic head sits in the curvature of the
duodenum. It's tail lies close to the hilum of the spleen, a site of potential injury
during splenectomy.
Relations
Posterior to the pancreas
Pancreatic head
Inferior vena cava
Common bile duct
Right and left renal veins
Superior mesenteric vein and artery
Pancreatic neck
Superior mesenteric vein, portal vein
Pancreatic bodyLeft renal vein
Crus of diaphragm
Psoas muscle
Adrenal gland
Kidney
Aorta
Pancreatic tail
Left kidney
Anterior to the pancreas
Pancreatic head

Pancreatic body
Pancreatic tail

1st part of the duodenum


Pylorus
Gastroduodenal artery
SMA and SMV(uncinate process)
Stomach
Duodenojejunal flexure
Splenic hilum

Superior to the pancreas


Coeliac trunk and its branches common hepatic artery and splenic artery
Grooves of the head of the pancreas
2nd and 3rd part of the duodenum
Arterial supply

Head: pancreaticoduodenal artery

Rest: splenic artery

Venous drainage

Head: superior mesenteric vein

Body and tail: splenic vein

Ampulla of Vater

Merge of pancreatic duct and common bile duct

Is an important landmark, halfway along the second part of the duodenum, that
marks the anatomical transition from foregut to midgut (also the site of
transition between regions supplied by coeliac trunk and SMA).

Image sourced from Wikipedia

Surgical occlusion of which of these structures, will result in the greatest reduction in
hepatic blood flow?
A. Portal vein
B. Common hepatic artery
C. Right hepatic artery
D. Coeliac axis
E. Left hepatic artery
The portal vein transports 70% of the blood supply to the liver, while the hepatic
artery provides 30%. The portal vein contains the products of digestion. The arterial
and venous blood is dispersed by sinusoids to the central veins of the liver lobules;
these drain into the hepatic veins and then into the IVC. The caudate lobe drains
directly into the IVC rather than into other hepatic veins.
Liver

Structure of the liver


Right lobe
Supplied by right hepatic artery

Contains Couinard segments V to VIII (-/+Sg I)

Supplied by the left hepatic artery

Contains Couinard segments II to IV (+/- Sg1)

Part of the right lobe anatomically, functionally is part of the


left

Couinard segment IV

Porta hepatis lies behind

On the right lies the gallbladder fossa

On the left lies the fossa for the umbilical vein

Supplied by both right and left hepatic arteries

Couinard segment I

Lies behind the plane of the porta hepatis

Anterior and lateral to the inferior vena cava

Bile from the caudate lobe drains into both right and left
hepatic ducts

Left lobe

Quadrate
lobe

Caudate lobe

Detailed knowledge of Couinard segments is not required for MRCS Part A

Between the liver lobules are portal canals which contain the portal triad:
Hepatic Artery, Portal Vein, tributary of Bile Duct.

Relations of the liver


Anterior
Diaphragm

Postero inferiorly
Oesophagus

Xiphoid process

Stomach
Duodenum
Hepatic flexure of colon
Right kidney
Gallbladder
Inferior vena cava

Porta hepatis
Location Postero inferior surface, it joins nearly at right angles with the left sagittal
fossa, and separates the caudate lobe behind from the quadrate lobe in
front
Transmits
Common hepatic duct

Hepatic artery

Portal vein

Sympathetic and parasympathetic nerve fibres

Lymphatic drainage of the liver (and nodes)

Ligaments
Falciform ligament

2 layer fold peritoneum from the umbilicus to anterior


liver surface

Contains ligamentum teres (remnant umbilical vein)

On superior liver surface it splits into the coronary and


left triangular ligaments

Ligamentum teres Joins the left branch of the portal vein in the porta hepatis
Ligamentum
Remnant of ductus venosus
venosum
Arterial supply

Hepatic artery

Venous

Hepatic veins

Portal vein

Nervous supply

Sympathetic and parasympathetic trunks of coeliac plexus

A 23 year old man presents with appendicitis. A decision is made to


perform an appendicectomy. The operation commences with a 5cm
incision centered on McBurneys point. Which of the following structures
will be encountered first during the dissection?
A External oblique aponeurosis
.
B Internal oblique muscle
.
C Transversalis fascia
.
D Rectus sheath
.
E. Peritoneum

The external oblique will be encountered first in this location. The rectus
sheath lies more medially.
The external oblique muscle is the most superficial of the abdominal wall
muscles. It originates from the 5th to 12th ribs and passes inferomedially
to insert into the linea alba, pubic tubercle and anterior half of the iliac
crest. It is innervated by the thoracoabdominal nerves (T7-T11) and sub
costal nerves.
Abdominal wall
The 2 main muscles of the abdominal wall are the rectus abdominis
(anterior) and the quadratus lumborum (posterior).
The remaining abdominal wall consists of 3 muscular layers. Each muscle
passes from the lateral aspect of the quadratus lumborum posteriorly to
the lateral margin of the rectus sheath anteriorly. Each layer is muscular
posterolaterally and aponeurotic anteriorly.

Image sourced from Wikipedia

Muscles of abdominal wall


External
Lies most superficially
oblique

Internal
oblique

Transversu
s
abdominis

Originates from 5th to 12th ribs

Inserts into the anterior half of the outer aspect of the


iliac crest, linea alba and pubic tubercle

More medially and superiorly to the arcuate line, the


aponeurotic layer overlaps the rectus abdominis
muscle

The lower border forms the inguinal ligament

The triangular expansion of the medial end of the


inguinal ligament is the lacunar ligament.

Arises from the thoracolumbar fascia, the anterior 2/3


of the iliac crest and the lateral 2/3 of the inguinal
ligament

The muscle sweeps upwards to insert into the


cartilages of the lower 3 ribs

The lower fibres form an aponeurosis that runs from


the tenth costal cartilage to the body of the pubis

At its lowermost aspect it joins the fibres of the


aponeurosis of transversus abdominis to form the
conjoint tendon.

Innermost muscle

Arises from the inner aspect of the costal cartilages of


the lower 6 ribs , from the anterior 2/3 of the iliac crest
and lateral 1/3 of the inguinal ligament

Its fibres run horizontally around the abdominal wall


ending in an aponeurosis. The upper part runs
posterior to the rectus abdominis. Lower down the
fibres run anteriorly only.

The rectus abdominis lies medially running from the


pubic crest and symphysis to insert into the xiphoid
process and 5th, 6th and 7th costal cartilages. The
muscles lies in a aponeurosis as described above.

Nerve supply: anterior primary rami of T7-12

Surgical notes
During abdominal surgery it is usually necessary to divide either the
muscles or their aponeuroses. During a midline laparotomy it is desirable
to divide the aponeurosis. This will leave the rectus sheath intact above
the arcuate line and the muscles intact below it. Straying off the midline
will often lead to damage to the rectus muscles, particularly below the
arcuate line where they may often be in close proximity to each other.

A 23 year old man is undergoing an inguinal hernia repair. The surgeons mobilise the
spermatic cord and place it in a hernia ring. A small slender nerve is identified
superior to the cord. Which of the following nerves is it most likely to be?
A. Iliohypogastric nerve
B. Pudendal nerve
C. Femoral branch of the genitofemoral nerve
D. Ilioinguinal nerve
E. Obturator nerve
The ilioinguinal nerve passes through the inguinal canal and is the nerve most
commonly identified during hernia surgery. The genitofemoral nerve splits into two
branches, the genital branch passes through the inguinal canal within the cord
structures. The femoral branch of the genitofemoral nerve enters the thigh posterior to
the inguinal ligament, lateral to the femoral artery. The iliohypogastric nerve pierces
the external oblique aponeurosis above the superficial inguinal ring.
Ilioinguinal nerve
Arises from the first lumbar ventral ramus with the iliohypogastric nerve. It passes
inferolaterally through the substance of psoas major and over the anterior surface of
quaratus lumborum. It pierces the internal oblique muscle and passes deep to the
aponeurosis of the external oblique muscle. It enters the inguinal canal and then
passes through the superficial inguinal ring to reach the skin.

Branches

To supply those muscles of the abdominal wall through which it passes.

Skin and fascia over the pubic symphysis, superomedial part of the femoral
triangle, surface of the scrotum, root and dorsum of penis or labum majus in
females.

A 45 year old man is undergoing a low anterior resection for a carcinoma of the
rectum. Which of the following fascial structures will need to be divided to mobilise
the mesorectum from the sacrum and coccyx?
A.

Denonvilliers fascia

B.

Colles fascia

C.

Sibsons fascia

D.

Waldeyers fascia

E.

None of the above

Fascial layers surrounding the rectum:

Anteriorly lies the fascia of Denonvilliers

Posteriorly lies Waldeyers fascia

Waldeyers fascia separates the mesorectum from the sacrum and will need to be
divided.
Rectum
The rectum is approximately 12 cm long. It is a capacitance organ. It has both intra
and extraperitoneal components. The transition between the sigmoid colon is marked
by the disappearance of the tenia coli.The extra peritoneal rectum is surrounded by
mesorectal fat that also contains lymph nodes. This mesorectal fatty layer is removed
surgically during rectal cancer surgery (Total Mesorectal Excision). The fascial layers
that surround the rectum are important clinical landmarks, anteriorly lies the fascia of
Denonvilliers. Posteriorly lies Waldeyers fascia.
Extra peritoneal rectum

Posterior upper third

Posterior and lateral middle third

Whole lower third

Relations
Anteriorly (Males)

Anteriorly (Females)
Posteriorly
Laterally

Rectovesical pouch
Bladder
Prostate
Seminal vesicles
Recto-uterine pouch (Douglas)
Cervix
Vaginal wall
Sacrum
Coccyx
Middle sacral artery
Levator ani
Coccygeus

Arterial supply
Superior rectal artery
Venous drainage
Superior rectal vein
Lymphatic drainage

Mesorectal lymph nodes (superior to dentate line)

Internal iliac and then para-aortic nodes

Inguinal nodes (inferior to dentate line)

A 73 year old lady is admitted with acute mesenteric ischaemia. A CT


angiogram is performed and a stenotic lesion is noted at the origin of the
superior mesenteric artery. At which of the following levels does this
branch from the aorta?
A L1
.
B L2
.
C L3
.

D L4
.
E. L5

Theme from January 2012 Exam


The SMA leaves the aorta at L1. It passes under the neck of the pancreas
prior to giving its first branch the inferior pancreatico-duodenal artery.
Superior mesenteric artery

Branches off aorta at L1

Supplies small bowel from duodenum (distal to ampulla of vater)


through to mid transverse colon

Takes more oblique angle from aorta and thus more likely to recieve
emboli than coeliac axis

Relations of superior mesenteric artery


Superiorly
Neck of pancreas
Postero-inferiorly

Third part of duodenum


Uncinate process

Posteriorly

Left renal vein

Right

Superior mesenteric vein

Branches of the superior mesenteric artery

Inferior pancreatico-duodenal artery

Jejunal and ileal arcades

Ileo-colic artery

Right colic artery

Middle colic artery

Overview of SMA and branches

Image sourced from Wikipedia

72 year old man is undergoing an open abdominal aortic aneurysm repair.


The aneurysm is located in a juxtarenal location and surgical access to the
neck of aneurysm is difficult. Which of the following structures may be
divided to improve access?
A Cisterna chili
.
B Transverse colon
.
C Left renal vein
.
D Superior mesenteric artery
.
E. Coeliac axis

The left renal vein will be stretched over the neck of the anuerysm in this
location and is not infrequently divided. This adds to the nephrotoxic insult
of juxtarenal aortic surgery as a supra renal clamp is also often applied.
Deliberate division of the Cisterna Chyli will not improve access and will
result in a chyle leak. Division of the transverse colon will not help at all
and would result in a high risk of graft infection. Division of the SMA is
pointless for a juxtarenal procedure.

Abdominal aorta
Abdominal aortic topography
Origin

T12

Termination

L4

Posterior relations

L1-L4 Vertebral bodies

Anterior relations

Lesser omentum
Liver
Left renal vein
Inferior mesenteric vein
Third part of duodenum
Pancreas
Parietal peritoneum
Peritoneal cavity

Right lateral relations

Right crus of the diaphragm


Cisterna chyli
Azygos vein
IVC (becomes posterior distally)

Left lateral relations

4th part of duodenum


Duodenal-jejunal flexure
Left sympathetic trunk

The abdominal aorta

Image sourced from Wikipedia

Your consultant decides to perform an open inguinal hernia repair under local
anaesthesia. Which of the following dermatomal levels will require blockade?
A. T10
B. T12
C. T11
D. S1
E. S2
Theme from April 2012 Exam
Dermatomes
The common dermatomal levels and cutaneous nerves responsible for them is
illustrated below.

A 53 year old man presents with an inguinal hernia. Which of the following
surface landmarks may be used to identify the location of the deep
inguinal ring?
AMid point of the
. inguinal ligament
BThe mid inguinal
. point
CThe pubic tubercle
.

DThe medial edge


. of external oblique
E2cm supero
. medially to the
femoral artery
Surface marking of deep inguinal
ring = midpoint of inguinal
ligament
The surface markings of the deep inguinal ring are a commonly examined
topic and should be memorised. The surface marking is the midpoint of
the inguinal ligament. The mid inguinal point is the surface marking for the
femoral artery. The pubic tubercle marks the site of the superficial inguinal
ring.
Inguinal canal
Location

Above the inguinal ligament

The inguinal canal is 4 cm long

Boundaries of the inguinal canal


Floor
External oblique aponeurosis

Inguinal ligament

Lacunar ligament

Internal oblique

Transversus abdominis

Roof

Anterior wall

External oblique aponeurosis

Posterior wall

Transversalis fascia

Conjoint tendon

Laterally

Internal ring

Fibres of internal oblique

External ring

Conjoint tendon

Medially

Contents
Males Spermatic cord and
ilioinguinal nerve

As it passes through the canal the


spermatic cord has 3 coverings:

External spermatic fascia from


external oblique aponeurosis

Cremasteric fascia

Internal spermatic fascia

Femal Round ligament of uterus


es
and ilioinguinal nerve
Related anatomy of the inguinal region
The boundaries of Hesselbachs triangle are commonly tested and
illustrated below:

Image sourced from Wikipedia

The image below demonstrates the close relationship of the vessels to the
lower limb with the inguinal canal. A fact to be borne in mind when
repairing hernial defects in this region.

Image sourced from Wikipedia

At which level is the hilum of the left kidney located?

A. L1
B. L2
C. T12
D. T11
E. L3
Remember L1 ('left one') is the level of the hilum of the left kidney
Theme from April 2012 exam
This is commonly tested in the mrcs exam.
Renal anatomy
Each kidney is about 11cm long, 5cm wide and 3cm thick. They are located in a deep
gutter alongside the projecting verterbral bodies, on the anterior surface of psoas
major. In most cases the left kidney lies approximately 1.5cm higher than the right.
The upper pole of both kidneys approximates with the 11th rib (beware pneumothorax
during nephrectomy). On the left hand side the hilum is located at the L1 vertebral
level and the right kidney at level L1-2. The lower border of the kidneys is usually
alongside L3.
The table below shows the anatomical relations of the kidneys:
Relations
Relations Right Kidney
Posterior Quadratus lumborum, diaphragm,
psoas major, transversus abdominis
Anterior Hepatic flexure of colon
Superior Liver, adrenal gland

Left Kidney
Quadratus lumborum, diaphragm,
psoas major, transversus abdominis
Stomach, Pancreatic tail
Spleen, adrenal gland

Fascial covering
Each kidney and suprarenal gland is enclosed within a common and layer of investing
fascia that is derived from the transversalis fascia into anterior and posterior layers
(Gerotas fascia).
Renal structure
Kidneys are surrounded by an outer cortex and an inner medulla which usually
contains between 6 and 10 pyramidal structures. The papilla marks the innermost apex
of these. They terminate at the renal pelvis, into the ureter.
Lying in a hollow within the kidney is the renal sinus. This contains:
1. Branches of the renal artery
2. Tributaries of the renal vein
3. Major and minor calyces's
4. Fat

Structures at the renal hilum


The renal vein lies most anteriorly, then renal artery (it is an end artery) and the ureter
lies most posterior.
You are assisting in an open right adrenalectomy for a large adrenal adenoma. The
consultant is distracted and you helpfully pull the adrenal into the wound to improve
the view. Unfortunately this is followed by brisk bleeding. The vessel responsible for
this is most likely to be:
A. Portal vein
B. Phrenic vein
C. Right renal vein
D. Superior mesenteric vein
E. Inferior vena cava
It drains directly via a very short vessel. If the sutures are not carefully tied then it
may be avulsed off the IVC. An injury best managed using a Satinsky clamp and a 6/0
prolene suture.
Adrenal gland anatomy
Anatomy
Location
Relationships of the
right adrenal
Relationships of the
left adrenal
Arterial supply
Venous drainage of
the right adrenal
Venous drainage of
the left adrenal

Superomedially to the upper pole of each kidney


Diaphragm-Posteriorly, Kidney-Inferiorly, Vena CavaMedially, Hepato-renal pouch and bare area of the liverAnteriorly
Crus of the diaphragm-Postero- medially, Pancreas and
splenic vessels-Inferiorly, Lesser sac and stomach-Anteriorly
Superior adrenal arteries- from inferior phrenic artery,
Middle adrenal arteries - from aorta, Inferior adrenal arteries
-from renal arteries
Via one central vein directly into the IVC
Via one central vein into the left renal vein

An enthusiastic surgical registrar undertakes his first solo splenectomy. The operation
is far more difficult than anticipated and the registrar leaves a tube drain to the splenic
bed at the end of the procedure. Over the following 24 hours approximately 500ml of
clear fluid has entered the drain. Biochemical testing of the fluid is most likely to
reveal:
A. Elevated creatinine

B. Elevated triglycerides
C. Elevated glucagon
D. Elevated amylase
E. None of the above
During splenectomy the tail of the pancreas may be damaged. The pancreatic duct
will then drain into the splenic bed, amylase is the most likely biochemical finding.
Glucagon is not secreted into the pancreatic duct.
Splenic anatomy
The spleen is the largest lymphoid organ in the body. It is an intraperitoneal organ, the
peritoneal attachments condense at the hilum where the vessels enter the spleen. Its
blood supply is from the splenic artery (derived from the coeliac axis) and the splenic
vein (which is joined by the IMV and unites with the SMV).

Embryology: derived from mesenchymal tissue

Shape: clenched fist

Position: below 9th-12th ribs

Weight: 75-150g

Relations

Superiorly- diaphragm

Anteriorly- gastric impression

Posteriorly- kidney

Inferiorly- colon

Hilum: tail of pancreas and splenic vessels

Forms apex of lesser sac (containing short gastric vessels)

A 56 year old lady is referred to the colorectal clinic with symptoms of pruritus ani.
On examination a polypoidal mass is identified inferior to the dentate line. A biopsy
confirms squamous cell carcinoma. To which of the following lymph node groups will
the lesion potentially metastasise?

A. Internal iliac
B. External iliac
C. Mesorectal
D. Inguinal
E. None of the above
Theme from September 2011 Exam
Theme from April 2012 Exam
Lesions distal to the dentate line drain to the inguinal nodes. Occasionally this will
result in the need for a block dissection of the groin.
Rectum
The rectum is approximately 12 cm long. It is a capacitance organ. It has both intra
and extraperitoneal components. The transition between the sigmoid colon is marked
by the disappearance of the tenia coli.The extra peritoneal rectum is surrounded by
mesorectal fat that also contains lymph nodes. This mesorectal fatty layer is removed
surgically during rectal cancer surgery (Total Mesorectal Excision). The fascial layers
that surround the rectum are important clinical landmarks, anteriorly lies the fascia of
Denonvilliers. Posteriorly lies Waldeyers fascia.
Extra peritoneal rectum

Posterior upper third

Posterior and lateral middle third

Whole lower third

Relations
Anteriorly (Males)

Anteriorly (Females)
Posteriorly
Laterally

Rectovesical pouch
Bladder
Prostate
Seminal vesicles
Recto-uterine pouch (Douglas)
Cervix
Vaginal wall
Sacrum
Coccyx
Middle sacral artery
Levator ani
Coccygeus

Arterial supply
Superior rectal artery
Venous drainage
Superior rectal vein
Lymphatic drainage

Mesorectal lymph nodes (superior to dentate line)

Internal iliac and then para-aortic nodes

Inguinal nodes (inferior to dentate line)

A 72 year old man develops a hydrocele which is being surgically managed. As part
of the procedure the surgeons divide the tunica vaginalis. From which of the
following is this structure derived?
A. Peritoneum
B. External oblique aponeurosis
C. Internal oblique aponeurosis
D. Transversalis fascia
E. Rectus sheath
The tunica vaginalis is derived from peritoneum, it secretes the fluid that fills the
hydrocele cavity.
Scrotal and testicular anatomy
Spermatic cord
Formed by the vas deferens and is covered by the following structures:
Layer
Origin
Internal spermatic fascia
Transversalis fascia
Cremasteric fascia
From the fascial coverings of internal oblique
External spermatic fascia
External oblique aponeurosis
Contents of the cord
Vas deferens
Testicular artery
Artery of vas deferens
Cremasteric artery

Transmits sperm and accessory gland


secretions
Branch of abdominal aorta supplies testis and
epididymis
Arises from inferior vesical artery
Arises from inferior epigastic artery

Pampiniform plexus
Sympathetic nerve fibres
Genital branch of the
genitofemoral nerve
Lymphatic vessels

Venous plexus, drains into right or left


testicular vein
Lie on arteries, the parasympathetic fibres lie
on the vas
Supplies cremaster
Drain to lumbar and para-aortic nodes

Scrotum

Composed of skin and closely attached dartos fascia.

Arterial supply from the anterior and posterior scrotal arteries

Lymphatic drainage to the inguinal lymph nodes

Parietal layer of the tunica vaginalis is the innermost layer

Testes

The testes are surrounded by the tunica vaginalis (closed peritoneal sac). The
parietal layer of the tunica vaginalis adjacent to the internal spermatic fascia.

The testicular arteries arise from the aorta immediately inferiorly to the renal
arteries.

The pampiniform plexus drains into the testicular veins, the left drains into the
left renal vein and the right into the inferior vena cava.

Lymphatic drainage is to the para-aortic nodes.

A 43 year old lady is donating her left kidney to her sister and the surgeons are
harvesting the left kidney. Which of the following structures will lie most anteriorly at
the hilum of the left kidney?
A. Left renal artery
B. Left renal vein
C. Left ureter
D. Left ovarian vein
E. Left ovarian artery
The renal veins lie most anteriorly, then artery and ureter lies posteriorly.

Renal arteries

The right renal artery is longer than the left renal artery

The renal vein/artery/pelvis enter the kidney at the hilum

Relations

Right:

Anterior- IVC, right renal vein, the head of the pancreas, and the descending part of
the duodenum.

Left:

Anterior- left renal vein, the tail of the pancreas.


Branches

The renal arteries are direct branches off the aorta (upper border of L2)

In 30% there may be accessory arteries (mainly left side). Instead of entering
the kidney at the hilum, they usually pierce the upper or lower part of the
organ.

Before reaching the hilum of the kidney, each artery divides into four or five
segmental branches (renal vein anterior and ureter posterior); which then
divide within the sinus into lobar arteries supplying each pyramid and cortex.

Each vessel gives off some small inferior suprarenal branches to the
suprarenal gland, the ureter, and the surrounding cellular tissue and muscles.

A 56 year old lady is due to undergo a left hemicolectomy for carcinoma of the
splenic flexure. The surgeons decide to perform a high ligation of the inferior
mesenteric vein. Into which of the following does this structure usually drain?
A. Portal vein
B. Inferior vena cava
C. Left renal vein
D. Left iliac vein

E. Splenic vein
Beware of ureteric injury in colonic surgery.
The IMV drains into the splenic vein.
Left colon
Position

As the left colon passes inferiorly its posterior aspect becomes extraperitoneal,
and the ureter and gonadal vessels are close posterior relations that may
become involved in disease processes

At a level of L3-4 (variable) the left colon becomes the sigmoid colon and
wholly intraperitoneal once again

The sigmoid colon is a highly mobile structure and may even lie of the right
side of the abdomen

It passes towards the midline, the taenia blend and this marks the transition
between sigmoid colon and upper rectum.

Blood supply

Inferior mesenteric artery

However, the marginal artery (from the right colon) contributes and this
contribution becomes clinically significant when the IMA is divided surgically
(e.g. During AAA repair)

Which of the following statements relating to quadratus lumborum is


false?
A Causes flexion of the thoracic spine
.
B Causes the rib cage to be pulled down
.
C Innervated by anterior primary rami of T12 and L1-3
.
D Attached to the iliac crest
.

E. Inserts into the 12th rib

The rectus abdominis causes flexion of the thoracic spine.


Attached to the medial iliac crest and iliolumbar ligament.
Inserts: 12th rib
Action: pulls the rib cage inferiorly. Lateral flexion.
Nerve supply: anterior primary rami of T12 and L1-3
Abdominal wall
The 2 main muscles of the abdominal wall are the rectus abdominis
(anterior) and the quadratus lumborum (posterior).
The remaining abdominal wall consists of 3 muscular layers. Each muscle
passes from the lateral aspect of the quadratus lumborum posteriorly to
the lateral margin of the rectus sheath anteriorly. Each layer is muscular
posterolaterally and aponeurotic anteriorly.

Image sourced from Wikipedia

Muscles of abdominal wall


External
Lies most superficially
oblique

Originates from 5th to 12th ribs

Inserts into the anterior half of the outer aspect of the


iliac crest, linea alba and pubic tubercle

More medially and superiorly to the arcuate line, the


aponeurotic layer overlaps the rectus abdominis
muscle

The lower border forms the inguinal ligament

The triangular expansion of the medial end of the


inguinal ligament is the lacunar ligament.

Internal
oblique

Transversu
s
abdominis

Arises from the thoracolumbar fascia, the anterior 2/3


of the iliac crest and the lateral 2/3 of the inguinal
ligament

The muscle sweeps upwards to insert into the


cartilages of the lower 3 ribs

The lower fibres form an aponeurosis that runs from


the tenth costal cartilage to the body of the pubis

At its lowermost aspect it joins the fibres of the


aponeurosis of transversus abdominis to form the
conjoint tendon.

Innermost muscle

Arises from the inner aspect of the costal cartilages of


the lower 6 ribs , from the anterior 2/3 of the iliac crest
and lateral 1/3 of the inguinal ligament

Its fibres run horizontally around the abdominal wall


ending in an aponeurosis. The upper part runs
posterior to the rectus abdominis. Lower down the
fibres run anteriorly only.

The rectus abdominis lies medially running from the


pubic crest and symphysis to insert into the xiphoid
process and 5th, 6th and 7th costal cartilages. The
muscles lies in a aponeurosis as described above.

Nerve supply: anterior primary rami of T7-12

Surgical notes
During abdominal surgery it is usually necessary to divide either the
muscles or their aponeuroses. During a midline laparotomy it is desirable
to divide the aponeurosis. This will leave the rectus sheath intact above
the arcuate line and the muscles intact below it. Straying off the midline
will often lead to damage to the rectus muscles, particularly below the
arcuate line where they may often be in close proximity to each other.

A 67 year old man is undergoing an angiogram for gastro intestinal bleeding. The
radiologist advances the catheter into the coeliac axis. At what spinal level does this
vessel typically arise from the aorta?
A. T10
B. L3

C. L4
D. T12
E. None of the above
The coeliac axis lies at T12, it takes an almost horizontal angle off the aorta. It has
three major branches.
Abdominal aortic branches
Branches
Level
Paired
Type
Inferior phrenic
T12 (Upper border)
Yes
Parietal
Coeliac
T12
No
Visceral
Superior mesenteric
L1
No
Visceral
Middle suprarenal
L1
Yes
Visceral
Renal
L1-L2
Yes
Visceral
Gonadal
L2
Yes
Visceral
Lumbar
L1-L4
Yes
Parietal
Inferior mesenteric
L3
No
Visceral
Median sacral
L4
No
Parietal
Common iliac
L4
Yes
Terminal
During a radical gastrectomy for carcinoma of the stomach the surgeons remove the
omentum. What is the main source of its blood supply?
A. Ileocolic artery
B. Superior mesenteric artery
C. Gastroepiploic artery
D. Middle colic artery
E. Inferior mesenteric artery
The vessels supplying the omentum are the omental branches of the right and left
gastro-epiploic arteries. The colonic vessels are not responsible for the arterial supply
to the omentum. The left gastro-epiploic artery is a branch of the splenic artery and
the right gastro-epiploic artery is a terminal branch of the gastroduodenal artery.
Omentum

The omentum is divided into two parts which invest the stomach. Giving rise
to the greater and lesser omentum. The greater omentum is attached to the
inferolateral border of the stomach and houses the gastro-epiploic arteries.

It is of variable size but is less well developed in children. This is important as


the omentum confers protection against visceral perforation (e.g.
Appendicitis).

Inferiorly between the omentum and transverse colon is one potential entry
point into the lesser sac.

Several malignant processes may involve the omentum of which ovarian


cancer is the most notable.

A 45 year old man has a long fermoral line inserted to provide CVP
measurements. The catheter passes from the common iliac vein into the
inferior vena cava. At which of the following vertebral levels will this
occur?
A L5
.
B L4
.
C S1
.
D L3
.
E. L2

The common iliac veins fuse with the IVC at L5.


Inferior vena cava
Origin

L5

Path

Left and right common iliac veins merge to form the IVC.

Passes right of midline

Paired segmental lumbar veins drain into the IVC throughout its
length

The right gonadal vein empties directly into the cava and the left
gonadal vein generally empties into the left renal vein.

The next major veins are the renal veins and the hepatic veins

Pierces the central tendon of diaphragm at T8

Right atrium

Image sourced from Wikipedia

Relations
Anterior Small bowel, first and third part of duodenum, head of pancreas,
ly
liver and bile duct, right common iliac artery, right gonadal
artery
Posteri Right renal artery, right psoas, right sympathetic chain, coeliac
orly
ganglion
Levels
Level Vein
T8

Hepatic vein, inferior phrenic vein, pierces diaphragm

L1

Suprarenal veins, renal vein

L2

Gonadal vein

L1-5

Lumbar veins

L5

Common iliac vein, formation of IVC

What is the nerve root value of the external urethral sphincter?


A. S4
B. S1, S2, S3
C. S2, S3, S4
D. L3, L4, L5
E. L5, S1, S2
Theme from April 2012 Exam
The external urethral sphincter is innervated by branches of the pudendal nerve,
therefore the root values are S2, S3, S4.
Urethral anatomy
Female urethra
The female urethra is shorter and more acutely angulated than the male urethra. It is
an extra-peritoneal structure and embedded in the endopelvic fascia. The neck of the
bladder is subjected to transmitted intra-abdominal pressure and therefore deficiency
in this area may result in stress urinary incontinence. Between the layers of the
urogenital diaphragm the female urethra is surrounded by the external urethral
sphincter, this is innervated by the pudendal nerve. It ultimately lies anterior to the
vaginal orifice.
Male urethra
In males the urethra is much longer and is divided into four parts.
Pre-prostatic
urethra

Extremely short and lies between the bladder and prostate gland.It
has a stellate lumen and is between 1 and 1.5cm long.Innervated by
sympathetic noradrenergic fibres, as this region is composed of
striated muscles bundles they may contract and prevent retrograde
ejaculation.
Prostatic
This segment is wider than the membranous urethra and contains
urethra
several openings for the transmission of semen (at the midpoint of
the urethral crest).
Membranous Narrowest part of the urethra and surrounded by external sphincter. It
urethra
traverses the perineal membrane 2.5cm postero-inferior to the
symphysis pubis.
Penile urethra Travels through the corpus songiosum on the underside of the penis.
It is the longest urethral segment.It is dilated at its origin as the
infrabulbar fossa and again in the gland penis as the navicular fossa.

The bulbo-urethral glands open into the spongiose section of the


urethra 2.5cm below the perineal membrane.
The urothelium is transitional in nature near to the bladder and becomes squamous
more distally.
A 32 year old man is undergoing a splenectomy. Division of which of the following
will be necessary during the procedure?
A. Left crus of diaphragm
B. Short gastric vessels
C. Gerotas fascia
D. Splenic flexure of colon
E. Marginal artery
During a splenectomy the short gastric vessels which lie within the gastrosplenic
ligament will need to be divided. The splenic flexure of the colon may need to be
mobilised. However, it will almost never need to be divided, as this is watershed area
that would necessitate a formal colonic resection in the event of division.
Splenic anatomy
The spleen is the largest lymphoid organ in the body. It is an intraperitoneal organ, the
peritoneal attachments condense at the hilum where the vessels enter the spleen. Its
blood supply is from the splenic artery (derived from the coeliac axis) and the splenic
vein (which is joined by the IMV and unites with the SMV).

Embryology: derived from mesenchymal tissue

Shape: clenched fist

Position: below 9th-12th ribs

Weight: 75-150g

Relations

Superiorly- diaphragm

Anteriorly- gastric impression

Posteriorly- kidney

Inferiorly- colon

Hilum: tail of pancreas and splenic vessels

Forms apex of lesser sac (containing short gastric vessels)

Two teenagers are playing with an airgun when one accidentally shoots his friend in
the abdomen. He is brought to the emergency department. On examination there is a
bullet entry point immediately to the right of the rectus sheath at the level of the 1st
lumbar vertebra. Which of the following structures is most likely to be injured by the
bullet?
A. Head of pancreas
B. Right ureter
C. Right adrenal gland
D. Fundus of the gallbladder
E. Gastric antrum
Theme from September 2011 Exam
The fundus of the gallbladder lies at this level and is the most superficially located
structure.
Levels
Transpyloric plane
Level of the body of L1

Pylorus stomach

Left kidney hilum (L1- left one!)

Right hilum of the kidney (1.5cm lower than the left)

Fundus of the gallbladder

Neck of pancreas

Duodenojejunal flexure

Superior mesenteric artery

Portal vein

Left and right colic flexure

Root of the transverse mesocolon

2nd part of the duodenum

Upper part of conus medullaris

Spleen

Can be identified by asking the supine patient to sit up without using their arms. The
plane is located where the lateral border of the rectus muscle crosses the costal
margin.
Anatomical planes
Subcostal plane
Intercristal plane
Intertubercular plane

Lowest margin of 10th costal cartilage


Level of body L4 (highest point of iliac crest)
Level of body L5

Common level landmarks


Inferior mesenteric artery
L3
Bifurcation of aorta into common iliac arteries L4
Formation of IVC
L5 (union of common iliac veins)
Diaphragm apertures
Vena cava T8

Oesophagus T10

Aortic hiatus T12

hich of the following anatomical planes separates the prostate from the rectum?
A. Sibsons fascia
B. Denonvilliers fascia
C. Levator ani muscle
D. Waldeyers fascia
E. None of the above
The Denonvilliers fascia separates the rectum from the prostate. Waldeyers fascia
separates the rectum from the sacrum
Prostate gland
The prostate gland is approximately the shape and size of a walnut and is located
inferior to the bladder. It is separated from the rectum by Denonvilliers fascia and its

blood supply is derived from the internal iliac vessels. The internal sphincter lies at
the apex of the gland and may be damaged during prostatic surgery, affected
individuals may complain of retrograde ejaculation.
MIDDLE RECTAL ARTERY
Summary of prostate gland
Arterial supply Inferior vesical artery (from internal iliac)
Venous drainage Prostatic venous plexus (to paravertebral veins)
INTERNAL ILIAC
Lymphatic
Internal iliac nodes
SUPERIOR
VESICULAR
AR
drainage
INFERIOR RECTAL
Innervation
Inferior hypogastric plexus
INTERNAL PUDENDAL AR
Dimensions
INTERNAL PUDENDA
Transverse diameter (4cm)
INFERIOR VESICULAR AR
INTERNAL ILIAC

AP diameter (2cm)

Height (3cm)

Posterior lobe: posterior to urethra

Median lobe: posterior to urethra, in between ejaculatory


ducts

Lateral lobes x 2

Isthmus

Peripheral zone: subcapsular portion of posterior prostate.


Most prostate cancers are here

Central zone

Transition zone

Stroma

Lobes

Zones

Relations
Anterior
Posterior
Lateral

Pubic symphysis
Prostatic venous plexus
Denonvilliers fascia
Rectum
Ejaculatory ducts
Venous plexus (lies on prostate)
Levator ani (immediately below the puboprostatic ligaments)

A 56 year old lady is undergoing an adrenalectomy for Conns syndrome. During the
operation the surgeon damages the middle adrenal artery and haemorrhage ensues.
From which of the following structures does this vessel originate?
A. Aorta
B. Renal artery
C. Splenic artery
D. Coeliac axis
E. Superior mesenteric artery
The middle adrenal artery is usually a branch of the aorta, the lower adrenal artery
typically arises from the renal vessels.
Adrenal gland anatomy
Anatomy
Location
Relationships of the
right adrenal
Relationships of the
left adrenal
Arterial supply
Venous drainage of
the right adrenal
Venous drainage of

Superomedially to the upper pole of each kidney


Diaphragm-Posteriorly, Kidney-Inferiorly, Vena CavaMedially, Hepato-renal pouch and bare area of the liverAnteriorly
Crus of the diaphragm-Postero- medially, Pancreas and
splenic vessels-Inferiorly, Lesser sac and stomach-Anteriorly
Superior adrenal arteries- from inferior phrenic artery,
Middle adrenal arteries - from aorta, Inferior adrenal arteries
-from renal arteries
Via one central vein directly into the IVC
Via one central vein into the left renal vein

the left adrenal


24 year old man falls and lands astride a manhole cover. He suffers from a injury to
the anterior bulbar urethra. Where will the extravasated urine tend to collect?
A. Lesser pelvis
B. Connective tissue of the scrotum
C. Deep perineal space
D. Ischiorectal fossa
E. Posterior abdominal wall
This portion of the urethra is contained between the perineal membrane an the
membranous layer of the superficial fascia. As these are densely adherent to the
ischiopubic rami, extravasated urine cannot pass posteriorly because the 2 layers are
continuous around the superficial transverse perineal muscles.
Lower genitourinary tract trauma

Most bladder injuries occur due to blunt trauma

85% associated with pelvic fractures

Easily overlooked during assessment in trauma

Up to 10% of male pelvic fractures are associated with urethral or bladder


injuries

Types of injury
Urethral injury

Mainly in males

Blood at the meatus (50% cases)

There are 2 types:

i.Bulbar rupture
- most common
- straddle type injury e.g. bicycles
- triad signs: urinary retention, perineal haematoma, blood
at the meatus
ii. Membranous rupture

- can be extra or intraperitoneal


- commonly due to pelvic fracture
- Penile or perineal oedema/ hematoma
- PR: prostate displaced upwards (beware co-existing
retroperitoneal haematomas as they may make examination
difficult)
- Investigation: ascending urethrogram
- Management: suprapubic catheter (surgical placement,
not percutaneously)
External genitalia
injuries (i.e., the penis
and the scrotum)

Secondary to injuries caused by penetration, blunt


trauma, continence- or sexual pleasure-enhancing
devices, and mutilation

rupture is intra or extraperitoneal

presents with haematuria or suprapubic pain

history of pelvic fracture and inability to void:


always suspect bladder or urethral injury

inability to retrieve all fluid used to irrigate the


bladder through a Foley catheter indicates bladder
injury

investigation- IVU or cystogram

management: laparotomy if intraperitoneal,


conservative if extraperitoneal

Bladder injury

A 73 year old man presents with symptoms of mesenteric ischaemia. As


part of his diagnostic work up a diagnostic angiogram is performed .The
radiologist is attempting to cannulate the coeliac axis from the aorta. At
which of the following vertebral levels does this is usually originate?
A T10
.
B L2
.
C L3
.

D T8
.
E T12
.
Coeliac trunk branches:
Left Hand Side (LHS)
Left gastric
Hepatic
Splenic
The coeliac axis branches off the aorta at T12.
Coeliac axis
The coeliac axis has three main branches.

Left gastric

Hepatic: branches-Right Gastric, Gastroduodenal, Right


Gastroepiploic, Superior Pancreaticoduodenal, Cystic.

Splenic: branches- Pancreatic, Short Gastric, Left Gastroepiploic

It occasionally gives off one of the inferior phrenic arteries.

Image sourced from Wikipedia

Relations
Anteriorly

Lesser omentum

Right

Right coeliac ganglion and caudate process of liver

Left

Left coeliac ganglion and gastric cardia

Inferiorly

Upper border of pancreas and renal vein

During a liver resection a surgeon performs a pringles manoeuvre to control bleeding.


Which of the following structures will lie posterior to the epiploic foramen at this
level?
A. Hepatic artery
B. Cystic duct
C. Greater omentum
D. Superior mesenteric artery
E. Inferior vena cava
Bleeding from liver trauma or a difficult cholecystectomy can be controlled with a
vascular clamp applied at the epiploic foramen.
The epiploic foramen has the following boundaries:
Anteriorly (in the free edge of the lesser omentum): Bile duct to the right, portal vein
behind and hepatic artery to the left.
PosteriorlyInferior vena cava
Inferiorly1st part of the duodenum
SuperiorlyCaudate process of the liver
Liver
Structure of the liver
Right lobe
Supplied by right hepatic artery

Contains Couinard segments V to VIII (-/+Sg I)

Supplied by the left hepatic artery

Contains Couinard segments II to IV (+/- Sg1)

Left lobe

Quadrate
lobe

Part of the right lobe anatomically, functionally is part of the


left

Couinard segment IV

Porta hepatis lies behind

On the right lies the gallbladder fossa

On the left lies the fossa for the umbilical vein

Supplied by both right and left hepatic arteries

Couinard segment I

Lies behind the plane of the porta hepatis

Anterior and lateral to the inferior vena cava

Bile from the caudate lobe drains into both right and left
hepatic ducts

Caudate lobe

Detailed knowledge of Couinard segments is not required for MRCS Part A

Between the liver lobules are portal canals which contain the portal triad:
Hepatic Artery, Portal Vein, tributary of Bile Duct.

Relations of the liver


Anterior
Diaphragm
Xiphoid process

Postero inferiorly
Oesophagus
Stomach
Duodenum
Hepatic flexure of colon
Right kidney
Gallbladder
Inferior vena cava

Porta hepatis
Location Postero inferior surface, it joins nearly at right angles with the left sagittal
fossa, and separates the caudate lobe behind from the quadrate lobe in
front

Transmits

Common hepatic duct

Hepatic artery

Portal vein

Sympathetic and parasympathetic nerve fibres

Lymphatic drainage of the liver (and nodes)

Ligaments
Falciform ligament

2 layer fold peritoneum from the umbilicus to anterior


liver surface

Contains ligamentum teres (remnant umbilical vein)

On superior liver surface it splits into the coronary and


left triangular ligaments

Ligamentum teres Joins the left branch of the portal vein in the porta hepatis
Ligamentum
Remnant of ductus venosus
venosum
Arterial supply

Hepatic artery

Venous

Hepatic veins

Portal vein

Nervous supply

Sympathetic and parasympathetic trunks of coeliac plexus

Which of the following is not considered a major branch of the descending thoracic
aorta?
A. Bronchial artery

B. Mediastinal artery
C. Inferior thyroid artery
D. Posterior intercostal artery
E. Oesophageal artery
The inferior thyroid artery is usually derived from the thyrocervical trunk, a branch of
the subclavian artery.
Thoracic aorta
Origin
T4
Terminates T12
Relations

Anteriorly (from top to bottom)-root of the left lung, the


pericardium, the oesophagus, and the diaphragm

Posteriorly-vertebral column, azygos vein

Right- hemiazygos veins, thoracic duct

Left- left pleura and lung

Lateral segmental branches: Posterior intercostal arteries

Lateral visceral: Bronchial arteries supply bronchial walls and


lung excluding the alveoli

Midline branches: Oesophageal arteries

Branches

During a right hemicolectomy the caecum is mobilised. As the bowel is retracted


medially a vessel is injured, posterior to the colon. Which of the following is the most
likely vessel?
A. Right colic artery
B. Inferior vena cava
C. Aorta
D. External iliac artery
E. Gonadal vessels

The gonadal vessels and ureter are important posterior relations that are at risk during
a right hemicolectomy.
Caecum
Location

Proximal right colon below the ileocaecal valve

Intraperitoneal

Psoas

Iliacus

Femoral nerve

Genitofemoral nerve

Gonadal vessels

Posterior relations

Anterior relations
Greater omentum
Arterial supply
Ileocolic artery
Lymphatic drainage Mesenteric nodes accompany the venous drainage

The caecum is the most distensible part of the colon and in complete large
bowel obstruction with a competent ileocaecal valve the most likely site of
eventual perforation.

A 66 year old man is undergoing a left nephro-ureterectomy. The surgeons remove the
ureter, which of the following is responsible for the blood supply to the proximal
ureter?
A. Branches of the renal artery
B. External iliac artery
C. Internal iliac artery
D. Direct branches from the aorta
E. Common iliac artery
The proximal ureter is supplied by branches from the renal artery. For the other
feeding vessels - see below.

Ureter

25-35 cm long

Muscular tube lined by transitional epithelium

Surrounded by thick muscular coat. Becomes 3 muscular layers as it crosses


the bony pelvis

Retroperitoneal structure overlying transverse processes L2-L5

Lies anterior to bifurcation of iliac vessels

Blood supply is segmental; renal artery, aortic branches, gonadal branches,


common iliac and internal iliac

Lies beneath the uterine artery

Which of the following is not a content of the porta hepatis?


A. Portal vein
B. Hepatic artery
C. Cystic duct
D. Hepatic lymph nodes
E. None of the above
The cystic duct lies outside the porta hepatis and is an important landmark in
laparoscopic cholecystectomy. The structures in the porta hepatis are:

Portal vein

Hepatic artery

Common hepatic duct

These structures divide immediately after or within the porta hepatis to supply the
functional left and right lobes of the liver.
The porta hepatis is also surrounded by lymph nodes, that may enlarge to produce
obstructive jaundice and parasympathetic nervous fibres that travel along vessels to
enter the liver.
Liver

Structure of the liver


Right lobe
Supplied by right hepatic artery

Contains Couinard segments V to VIII (-/+Sg I)

Supplied by the left hepatic artery

Contains Couinard segments II to IV (+/- Sg1)

Part of the right lobe anatomically, functionally is part of the


left

Couinard segment IV

Porta hepatis lies behind

On the right lies the gallbladder fossa

On the left lies the fossa for the umbilical vein

Supplied by both right and left hepatic arteries

Couinard segment I

Lies behind the plane of the porta hepatis

Anterior and lateral to the inferior vena cava

Bile from the caudate lobe drains into both right and left
hepatic ducts

Left lobe

Quadrate
lobe

Caudate lobe

Detailed knowledge of Couinard segments is not required for MRCS Part A

Between the liver lobules are portal canals which contain the portal triad:
Hepatic Artery, Portal Vein, tributary of Bile Duct.

Relations of the liver


Anterior
Diaphragm

Postero inferiorly
Oesophagus

Xiphoid process

Stomach
Duodenum
Hepatic flexure of colon
Right kidney
Gallbladder
Inferior vena cava

Porta hepatis
Location Postero inferior surface, it joins nearly at right angles with the left sagittal
fossa, and separates the caudate lobe behind from the quadrate lobe in
front
Transmits
Common hepatic duct

Hepatic artery

Portal vein

Sympathetic and parasympathetic nerve fibres

Lymphatic drainage of the liver (and nodes)

Ligaments
Falciform ligament

2 layer fold peritoneum from the umbilicus to anterior


liver surface

Contains ligamentum teres (remnant umbilical vein)

On superior liver surface it splits into the coronary and


left triangular ligaments

Ligamentum teres Joins the left branch of the portal vein in the porta hepatis
Ligamentum
Remnant of ductus venosus
venosum
Arterial supply

Hepatic artery

Venous

Hepatic veins

Portal vein

Nervous supply

Sympathetic and parasympathetic trunks of coeliac plexus

A man is stabbed in the abdomen during a fight. He is brought to the emergency


department. On examination there is a laceration in the anterior abdominal wall
immediately lateral to the left rectus abdominis muscle on a level with the upper
border of the first lumbar vertebra. Which of the following structures is most likely to
have been injured?
A. Head of the pancreas
B. Gastric antrum
C. Spleen
D. Right lobe of the liver
E. Superior mesenteric artery
The spleen is the most likely target in this instance.
Levels
Transpyloric plane
Level of the body of L1

Pylorus stomach

Left kidney hilum (L1- left one!)

Right hilum of the kidney (1.5cm lower than the left)

Fundus of the gallbladder

Neck of pancreas

Duodenojejunal flexure

Superior mesenteric artery

Portal vein

Left and right colic flexure

Root of the transverse mesocolon

2nd part of the duodenum

Upper part of conus medullaris

Spleen

Can be identified by asking the supine patient to sit up without using their arms. The
plane is located where the lateral border of the rectus muscle crosses the costal
margin.
Anatomical planes
Subcostal plane
Intercristal plane
Intertubercular plane

Lowest margin of 10th costal cartilage


Level of body L4 (highest point of iliac crest)
Level of body L5

Common level landmarks


Inferior mesenteric artery
L3
Bifurcation of aorta into common iliac arteries L4
Formation of IVC
L5 (union of common iliac veins)
Diaphragm apertures
Vena cava T8

Oesophagus T10

Aortic hiatus T12

A surgeon is due to perform a laparotomy for perforated duodenal ulcer.


An upper midline incision is to be performed. Which of the following
structures is the incision most likely to divide?
A Rectus abdominis muscle
.
B External oblique muscle
.
C Linea alba
.
D Internal oblique muscle
.
E. None of the above

Theme from September 2011 Exam


Upper midline abdominal incisions will involve the division of the linea
alba. Division of muscles will not usually improve access in this approach
and they would not be routinely encountered during this incision.
Abdominal incisions
Theme in January 2012 exam
Midline
incision

Paramedian
incision

Commonest approach to the abdomen

Structures divided: linea alba, transversalis fascia,


extraperitoneal fat, peritoneum (avoid falciform
ligament above the umbilicus)

Bladder can be accessed via an extraperitoneal


approach through the space of Retzius

Parallel to the midline (about 3-4cm)

Structures divided/retracted: anterior rectus sheath,


rectus (retracted), posterior rectus sheath,
transversalis fascia, extraperitoneal fat, peritoneum

Incision is closed in layers

Similar location to paramedian but rectus displaced


medially (and thus denervated)

Now seldom used

Battle

Kocher's

Incision under right subcostal margin e.g. Cholecystectomy


(open)

Lanz

Incision in right iliac fossa e.g. Appendicectomy

Gridiron

Oblique incision centered over McBurneys point- usually


appendicectomy (less cosmetically acceptable than Lanz

Gable

Rooftop incision

Pfannenstie Transverse supra pubic, primarily used to access pelvic


l's
organs
McEvedy's

Groin incision e.g. Emergency repair strangulated femoral

hernia
Rutherford Extraperitoneal approach to left or right lower quadrants.
Morrison
Gives excellent access to iliac vessels and is the approach
of choice for first time renal transplantation.

Image sourced from Wikipedia

A 59 year old man is undergoing an extended right hemicolectomy for a carcinoma of


the hepatic flexure of the colon. The surgeons divide the middle colonic vein close to
its origin. Into which of the following structures does this vessel primarily drain?
A. Superior mesenteric vein
B. Portal vein
C. Inferior mesenteric vein
D. Inferior vena cava
E. Ileocolic vein
The middle colonic vein drains into the SMV, if avulsed during mobilisation then
dramatic haemorrhage can occur and be difficult to control.
Transverse colon

The right colon undergoes a sharp turn at the level of the hepatic flexure to
become the transverse colon.

At this point it also becomes intraperitoneal.

It is connected to the inferior border of the pancreas by the transverse


mesocolon.

The greater omentum is attached to the superior aspect of the transverse colon
from which it can easily be separated. The mesentery contains the middle colic
artery and vein. The greater omentum remains attached to the transverse colon
up to the splenic flexure. At this point the colon undergoes another sharp turn.

Relations
Superior Liver and gall-bladder, the greater curvature of the stomach, and the lower
end of the spleen
Inferior Small intestine
Anterior Greater omentum
Posterior From right to left with the descending portion of the duodenum, the head of
the pancreas, convolutions of the jejunum and ileum, spleen
A 23 year old man is stabbed in the chest approximately 10cm below the right nipple.
In the emergency department a abdominal ultrasound scan shows a large amount of
intraperitoneal blood. Which of the following statements relating to the likely site of
injury is untrue?
A. Part of its posterior surface is devoid of peritoneum.
B. The quadrate lobe is contained within the functional right lobe.
C. Its nerve supply is from the coeliac plexus.
D. The hepatic flexure of the colon lies posterio-inferiorly.
E. The right kidney is closely related posteriorly.
The right lobe of the liver is the most likely site of injury. Therefore the answer is B as
the quadrate lobe is functionally part of the left lobe of the liver. The liver is largely
covered in peritoneum. Posteriorly there is an area devoid of peritoneum (the bare
area of the liver). The right lobe of the liver has the largest bare area (ans is larger
thant the left lobe).
Liver
Structure of the liver
Right lobe
Supplied by right hepatic artery

Contains Couinard segments V to VIII (-/+Sg I)

Left lobe

Quadrate
lobe

Supplied by the left hepatic artery

Contains Couinard segments II to IV (+/- Sg1)

Part of the right lobe anatomically, functionally is part of the


left

Couinard segment IV

Porta hepatis lies behind

On the right lies the gallbladder fossa

On the left lies the fossa for the umbilical vein

Supplied by both right and left hepatic arteries

Couinard segment I

Lies behind the plane of the porta hepatis

Anterior and lateral to the inferior vena cava

Bile from the caudate lobe drains into both right and left
hepatic ducts

Caudate lobe

Detailed knowledge of Couinard segments is not required for MRCS Part A

Between the liver lobules are portal canals which contain the portal triad:
Hepatic Artery, Portal Vein, tributary of Bile Duct.

Relations of the liver


Anterior
Diaphragm
Xiphoid process

Postero inferiorly
Oesophagus
Stomach
Duodenum
Hepatic flexure of colon
Right kidney
Gallbladder
Inferior vena cava

Porta hepatis
Location Postero inferior surface, it joins nearly at right angles with the left sagittal
fossa, and separates the caudate lobe behind from the quadrate lobe in
front
Transmits
Common hepatic duct

Hepatic artery

Portal vein

Sympathetic and parasympathetic nerve fibres

Lymphatic drainage of the liver (and nodes)

Ligaments
Falciform ligament

2 layer fold peritoneum from the umbilicus to anterior


liver surface

Contains ligamentum teres (remnant umbilical vein)

On superior liver surface it splits into the coronary and


left triangular ligaments

Ligamentum teres Joins the left branch of the portal vein in the porta hepatis
Ligamentum
Remnant of ductus venosus
venosum
Arterial supply

Hepatic artery

Venous

Hepatic veins

Portal vein

Nervous supply

Sympathetic and parasympathetic trunks of coeliac plexus

Which of the following nerves passes through the greater sciatic foramen and
innervates the perineum?
A.

Pudendal

B.

Sciatic

C.

Superior gluteal

D.

Inferior gluteal

E.

Posterior cutaneous nerve of the thigh

3 divisions of the pudendal nerve:

Rectal nerve

Perineal nerve

Dorsal nerve of penis/ clitoris

All these pass through the greater sciatic foramen.


The pudendal nerve innervates the perineum. It passes between piriformis and
coccygeus medial to the sciatic nerve.
Gluteal region
Gluteal muscles

Gluteus maximus: inserts to gluteal tuberosity of the femur and iliotibial tract

Gluteus medius: attach to lateral greater trochanter

Gluteus minimis: attach to anterior greater trochanter

All extend and abduct the hip

Deep lateral hip rotators

Piriformis

Gemelli

Obturator internus

Quadratus femoris

Nerves
Superior gluteal nerve (L5, S1)

Gluteus medius

Gluteus minimis

Tensor fascia lata

Inferior gluteal nerve


Gluteus maximus
Damage to the superior gluteal nerve will result in the patient developing a
Trendelenberg gait. Affected patients are unable to abduct the thigh at the hip joint.
During the stance phase, the weakened abductor muscles allow the pelvis to tilt down
on the opposite side. To compensate, the trunk lurches to the weakened side to attempt
to maintain a level pelvis throughout the gait cycle. The pelvis sags on the opposite
side of the lesioned superior gluteal nerve.
A 53 year old man is undergoing a left hemicolectomy for carcinoma of the
descending colon. From which embryological structure is this region of the
gastrointestinal tract derived?
A. Vitellino-intestinal duct
B. Hind gut
C. Mid gut
D. Fore gut
E. Woolffian duct
The left colon is embryologically part of the hind gut. Which accounts for its separate
blood supply via the IMA.
Colon anatomy
The colon is about 1.5m long although this can vary considerably.
Components:

Ascending colon

Transverse colon

Descending colon

Sigmoid colon

Arterial supply

Superior mesenteric artery and inferior mesenteric artery: linked by the marginal
artery.
Ascending colon: ileocolic and right colic arteries
Transverse colon: middle colic artery
Descending and sigmoid colon: left colic artery
Venous drainage
From regional veins (that accompany arteries) to superior and inferior mesenteric vein
Lymphatic drainage
Initially along nodal chains that accompany supplying arteries, then para-aortic nodes.
Embryology
Midgut- Second part of duodenum to 2/3 transverse colon
Hindgut- Distal 1/3 transverse colon to anus
Peritoneal location
The right and left colon are part intraperitoneal and part extraperitoneal. The sigmoid
and transverse colon are generally wholly intraperitoneal. This has implications for
the sequelae of perforations, which will tend to result in generalised peritonitis in the
wholly intra peritoneal segments.
You excitedly embark on your first laparoscopic cholecystectomy and during the
operation the anatomy of Calots triangle is more hostile than anticipated. Whilst
trying to apply a haemostatic clip you avulse the cystic artery. This is followed by
brisk haemorrhage. From which source is this most likely to originate ?
A. Right hepatic artery
B. Portal vein
C. Gastroduodenal artery
D. Liver bed
E. Common hepatic artery
The cystic artery is a branch of the right hepatic artery. There are recognised
variations in the anatomy of the blood supply to the gallbladder. However, the
commonest situation is for the cystic artery to branch from the right hepatic artery.
Gallbladder

Fibromuscular sac with capacity of 50ml

Columnar epithelium

Relations of the gallbladder

Anterior
Posterior

Laterally
Medially

Liver

Covered by peritoneum

Transverse colon

1st part of the duodenum

Right lobe of liver


Quadrate lobe of liver

Arterial supply
Cystic artery (branch of Right hepatic artery)
Venous drainage
Cystic vein
Nerve supply
Sympathetic- mid thoracic spinal cord, Parasympathetic- anterior vagal trunk
Common bile duct
Origin
Relations at
origin

Relations
distally

Confluence of cystic and common hepatic ducts

Medially - Hepatic artery

Posteriorly- Portal vein

Duodenum - anteriorly

Pancreas - medially and laterally

Right renal vein - posteriorly

Arterial supply Branches of hepatic artery and retroduodenal branches of


gastroduodenal artery
Calot's triangle
Medially
Inferiorly
Superiorly
Contents

Common hepatic duct


Cystic duct
Inferior edge of liver
Cystic artery

A 43 year old man suffers a pelvic fracture which is complicated by an injury to the
junction of the membranous urethra to the bulbar urethra. In which of the following
directions is the extravasated urine most likely to pass?

A. Posteriorly into extra peritoneal tissues


B. Laterally into the buttocks
C. Into the abdomen
D. Anteriorly into the connective tissues surrounding the scrotum
E. None of the above
The superficial perineal pouch is a compartment bounded superficially by the
superficial perineal fascia, deep by the perineal membrane (inferior fascia of the
urogenital diaphragm), and laterally by the ischiopubic ramus. It contains the crura of
the penis or clitoris, muscles, viscera, blood vessels, nerves, the proximal part of the
spongy urethra in males, and the greater vestibular glands in females.
When urethral rupture occurs as in this case the urine will tend to pass anteriorly
because the fascial condensations will prevent lateral and posterior passage of the
urine.
Urogenital triangle
The urogenital triangle is formed by the:

Ischiopubic inferior rami

Ischial tuberosities

A fascial sheet is attached to the sides, forming the inferior fascia of the urogenital
diaphragm.
It transmits the urethra in males and both the urethra and vagina in females. The
membranous urethra lies deep this structure and is surrounded by the external urethral
sphincter.
Superficial to the urogenital diaphragm lies the superficial perineal pouch. In males
this contains:

Bulb of penis

Crura of the penis

Superficial transverse perineal muscle

Posterior scrotal arteries

Posterior scrotal nerves

In females the internal pudendal artery branches to become the posterior labial arteries
in the superficial perineal pouch.
Which of the following statements relating to the gastroduodenal artery is untrue?
A. It runs posterior to the 1st part of the duodenum
B. It originates from the common hepatic artery
C. The bile duct is a close relation
D. The portal vein is closely related anteriorly
E. It terminates as the gastro-epiploic and superior pancreaticoduodenal
artery
The portal vein is located posteriorly and then separated from the artery by the
pancreas. The anatomy of this artery is important as it is a site of bleeding in
posteriorly sited duodenal ulcers. At laparotomy for bleeding from this vessel, the
relation of the bile duct should be remembered less it be caught inadvertently in a
stitch.
Gastroduodenal artery
Supplies
Pylorus, proximal part of the duodenum, and indirectly to the pancreatic head (via the
anterior and posterior superior pancreaticoduodenal arteries)
Path
Most commonly arises from the common hepatic artery of the coeliac trunk
Terminates by bifurcating into the right gastroepiploic artery and the superior
pancreaticoduodenal artery
Image showing stomach reflected superiorly to illustrate the relationship of the
gastroduodenal artery to the first part of the duodenum

Image sour

Through which of the following foramina does the genital branch of the genitofemoral
nerve exit the abdominal cavity?
A. Superficial inguinal ring
B. Sciatic notch
C. Obturator foramen
D. Femoral canal
E. Deep inguinal ring
The genitofemoral nerve divides into two branches as it approaches the inguinal
ligament. The genital branch passes anterior to the external iliac artery through the
deep inguinal ring into the inguinal canal. It communicates with the ilioinguinal nerve
in the inguinal canal (though this is seldom of clinical significance).
Genitofemoral nerve
Supplies
- Small area of the upper medial thigh
Path

- Arises from the first and second lumbar nerves


- Passes obliquely th/
rough Psoas major, and emerges from its medial border opposite the fibrocartilage
between the third and fourth lumbar vertebrae.
- It then descends on the surface of Psoas major, under cover of the peritoneum
- Divides into genital and femoral branches.
- The genital branch passes through the inguinal canal, within the spermatic cord, to
supply the skin overlying the skin and fascia of the scrotum. The femoral branch
enters the thigh posterior to the inguinal ligament, lateral to the femoral artery. It
supplies an area of skin and fascia over the femoral triangle.

It may be injured during abdominal or pelvic surgery, or during inguinal hernia


repairs.

A 63 year old lady is diagnosed as having an endometrial carcinoma arising from the
uterine body. To which nodal region will the tumour initially metastasise?
A. Para aortic nodes
B. Iliac lymph nodes
C. Inguinal nodes
D. Pres sacral nodes
E. Mesorectal lymph nodes
Theme from 2011 exam
Tumours of the uterine body will tend to spread to the iliac nodes initially. When the
tumour is expanding to cross different nodal margins this is of considerable clinical
significance if nodal clearance is performed during a Wertheims type hysterectomy.
Lymphatic drainage of the uterus and cervix

The uterine fundus has a lymphatic drainage that runs with the ovarian vessels
and may thus drain to the para-aortic nodes. Some drainage may also pass
along the round ligament to the inguinal nodes.

The body of the uterus drains through lymphatics contained within the broad
ligament to the iliac lymph nodes.

The cervix drains into three potential nodal stations; laterally through the
broad ligament to the external iliac nodes, along the lymphatics of the
uterosacral fold to the presacral nodes and posterolaterally along lymphatics
lying alongside the uterine vessels to the internal iliac nodes.

Which of the following structures is not located in the superficial perineal space in
females?
A. Posterior labial arteries
B. Pudendal nerve
C. Superficial transverse perineal muscle
D. Greater vestibular glands
E. None of the above
The pudendal nerve is located in the deep perineal space and then branches to
innervate more superficial structures.
Urogenital triangle
The urogenital triangle is formed by the:

Ischiopubic inferior rami

Ischial tuberosities

A fascial sheet is attached to the sides, forming the inferior fascia of the urogenital
diaphragm.
It transmits the urethra in males and both the urethra and vagina in females. The
membranous urethra lies deep this structure and is surrounded by the external urethral
sphincter.
Superficial to the urogenital diaphragm lies the superficial perineal pouch. In males
this contains:

Bulb of penis

Crura of the penis

Superficial transverse perineal muscle

Posterior scrotal arteries

Posterior scrotal nerves

In females the internal pudendal artery branches to become the posterior labial arteries
in the superficial perineal pouch.
Which of the following is not a branch of the hepatic artery?

A Pancreatic artery
.
B Cystic artery
.
C Right gastric artery
.
D Superior Pancreaticoduodenal artery
.
E. Right Gastroepiploic artery

The pancreatic artery is a branch of the splenic artery.


Coeliac axis
The coeliac axis has three main branches.

Left gastric

Hepatic: branches-Right Gastric, Gastroduodenal, Right


Gastroepiploic, Superior Pancreaticoduodenal, Cystic.

Splenic: branches- Pancreatic, Short Gastric, Left Gastroepiploic

It occasionally gives off one of the inferior phrenic arteries.

Image sourced from Wikipedia

Relations
Anteriorly

Lesser omentum

Right

Right coeliac ganglion and caudate process of liver

Left

Left coeliac ganglion and gastric cardia

Inferiorly

Upper border of pancreas and renal vein

A 56 year old man is undergoing a nephrectomy. The surgeons divide the renal artery.
At what level do these usually branch off the abdominal aorta?
A. T9
B. L2
C. L3
D. T10
E. L4
The renal arteries usually branch off the aorta on a level with L2.
Renal arteries

The right renal artery is longer than the left renal artery

The renal vein/artery/pelvis enter the kidney at the hilum

Relations

Right:

Anterior- IVC, right renal vein, the head of the pancreas, and the descending part of
the duodenum.

Left:

Anterior- left renal vein, the tail of the pancreas.


Branches

The renal arteries are direct branches off the aorta (upper border of L2)

In 30% there may be accessory arteries (mainly left side). Instead of entering
the kidney at the hilum, they usually pierce the upper or lower part of the
organ.

Before reaching the hilum of the kidney, each artery divides into four or five
segmental branches (renal vein anterior and ureter posterior); which then
divide within the sinus into lobar arteries supplying each pyramid and cortex.

Each vessel gives off some small inferior suprarenal branches to the suprarenal
gland, the ureter, and the surrounding cellular tissue and muscles.

A 22 year old man presents with appendicitis. At operation the appendix is retrocaecal
and difficult to access. Division of which of the following anatomical structures
should be undertaken?
A. Ileocolic artery
B. Mesentery of the caecum
C. Gonadal vessels
D. Lateral peritoneal attachments of the caecum
E. Right colic artery
The commonest appendiceal location is retrocaecal. Those struggling to find it at
operation should trace the tenia to the caecal pole where the appendix is located. If it
cannot be mobilised easily then division of the lateral caecal peritoneal attachments
(as for a right hemicolectomy) will allow caecal mobilisation and facilitate the
procedure.
Appendix

Location: Base of caecum.

Up to 10cm long.

Mainly lymphoid tissue (Hence mesenteric adenitis may mimic appendicitis).

Caecal taenia coli converge at base of appendix and form a longitudinal


muscle cover over the appendix. This convergence should facilitate its
identification at surgery if it is retrocaecal and difficult to find (which it can be
when people start doing appendicectomies!)

Arterial supply: Appendicular artery (branch of the ileocolic).

It is intra peritoneal.

McBurney's point

1/3 of the way along a line drawn from the Anterior Superior Iliac Spine to the
Umbilicus

6 Positions:

Retrocaecal 74%

Pelvic 21%

Postileal

Subcaecal

Paracaecal

Preileal

A 56 year old man is left impotent following an abdomino-perineal excision of the


colon and rectum. What is the most likely explanation?
A. Psychosexual issues related to an end colostomy
B. Damage to the sacral venous plexus during total mesorectal excision
C. Damage to the left ureter during sigmoid mobilisation
D. Damage to the hypogastric plexus during mobilisation of the inferior
mesenteric artery
E. Damage to the internal iliac artery during total mesorectal excision
Autonomic nerve injury is the most common cause.
Nerve lesions during surgery
A variety of different procedures carry the risk of iatrogenic nerve injury. These are
important not only from the patients perspective but also from a medicolegal
standpoint.
The following operations and their associated nerve lesions are listed here:

Posterior triangle lymph node biopsy and accessory nerve lesion.

Lloyd Davies stirrups and common peroneal nerve.

Thyroidectomy and laryngeal nerve.

Anterior resection of rectum and hypogastric autonomic nerves.

Axillary node clearance; long thoracic nerve, thoracodorsal nerve and


intercostobrachial nerve.

Inguinal hernia surgery and ilioinguinal nerve.

Varicose vein surgery- sural and saphenous nerves.

Posterior approach to the hip and sciatic nerve.

Carotid endarterectomy and hypoglossal nerve.

There are many more, with sound anatomical understanding of the commonly
performed procedures the incidence of nerve lesions can be minimised. They
commonly occur when surgeons operate in an unfamiliar tissue plane or by blind
placement of haemostats (not recommended).
1
2
Rate question: 3
4
5
A 73 year old man is due to undergo a radical prostatectomy for
carcinoma of the prostate gland. To which of the following lymph nodes
will the tumour drain primarily?
A Para aortic
.
B Internal iliac
.
C Superficial inguinal
.
D Meso rectal
.

E. None of the above

The prostate lymphatic drainage is primarily to the internal iliac nodes and
also the sacral nodes. Although internal iliac is the first site.
Prostate gland
The prostate gland is approximately the shape and size of a walnut and is
located inferior to the bladder. It is separated from the rectum by
Denonvilliers fascia and its blood supply is derived from the internal iliac
vessels. The internal sphincter lies at the apex of the gland and may be
damaged during prostatic surgery, affected individuals may complain of
retrograde ejaculation.
Summary of prostate gland
Arterial
supply

Inferior vesical artery (from internal iliac)

Venous
drainage

Prostatic venous plexus (to paravertebral veins)

Lymphatic
drainage

Internal iliac nodes

Innervation Inferior hypogastric plexus


Dimensions

Transverse diameter (4cm)

AP diameter (2cm)

Height (3cm)

Posterior lobe: posterior to urethra

Median lobe: posterior to urethra, in between


ejaculatory ducts

Lateral lobes x 2

Isthmus

Peripheral zone: subcapsular portion of posterior


prostate. Most prostate cancers are here

Lobes

Zones

Central zone

Transition zone

Stroma

Relations
Anterior Pubic symphysis
Prostatic venous plexus
Posterior Denonvilliers fascia
Rectum
Ejaculatory ducts
Lateral

Venous plexus (lies on prostate)


Levator ani (immediately below the puboprostatic ligaments)

Image sourced from Wikipedia

A 28 year old man has sustained a non salvageable testicular injury to his left testicle.
The surgeon decides to perform an orchidectomy and divides the left testicular artery.
From which of the following does this vessel originate?
A. Abdominal aorta
B. Internal iliac artery

C. Inferior epigastric artery


D. Inferior vesical artery
E. External iliac artery
The testicular artery is a branch of the abdominal aorta.
Scrotal and testicular anatomy
Spermatic cord
Formed by the vas deferens and is covered by the following structures:
Layer
Origin
Internal spermatic fascia
Transversalis fascia
Cremasteric fascia
From the fascial coverings of internal oblique
External spermatic fascia
External oblique aponeurosis
Contents of the cord
Vas deferens
Testicular artery
Artery of vas deferens
Cremasteric artery
Pampiniform plexus
Sympathetic nerve fibres
Genital branch of the
genitofemoral nerve
Lymphatic vessels

Transmits sperm and accessory gland


secretions
Branch of abdominal aorta supplies testis and
epididymis
Arises from inferior vesical artery
Arises from inferior epigastic artery
Venous plexus, drains into right or left
testicular vein
Lie on arteries, the parasympathetic fibres lie
on the vas
Supplies cremaster
Drain to lumbar and para-aortic nodes

Scrotum

Composed of skin and closely attached dartos fascia.

Arterial supply from the anterior and posterior scrotal arteries

Lymphatic drainage to the inguinal lymph nodes

Parietal layer of the tunica vaginalis is the innermost layer

Testes

The testes are surrounded by the tunica vaginalis (closed peritoneal sac). The
parietal layer of the tunica vaginalis adjacent to the internal spermatic fascia.

The testicular arteries arise from the aorta immediately inferiorly to the renal
arteries.

The pampiniform plexus drains into the testicular veins, the left drains into the
left renal vein and the right into the inferior vena cava.

Lymphatic drainage is to the para-aortic nodes.

A 44 year old man is stabbed in the back and the left kidney is injured. A
haematoma forms, which of the following fascial structures will contain the
haematoma?
A. Waldeyers fascia
B. Sibsons fascia
C. Bucks fascia
D. Gerotas fascia
E. Denonvilliers fascia

Renal anatomy
Each kidney is about 11cm long, 5cm wide and 3cm thick. They are located in
a deep gutter alongside the projecting verterbral bodies, on the anterior surface
of psoas major. In most cases the left kidney lies approximately 1.5cm higher
than the right. The upper pole of both kidneys approximates with the 11th rib
(beware pneumothorax during nephrectomy). On the left hand side the hilum
is located at the L1 vertebral level and the right kidney at level L1-2. The
lower border of the kidneys is usually alongside L3.
The table below shows the anatomical relations of the kidneys:

Relations
Relations Right Kidney
Left Kidney
Posterior Quadratus lumborum, diaphragm,
Quadratus lumborum, diaphragm,
psoas major, transversus abdominis psoas major, transversus abdominis
Anterior Hepatic flexure of colon
Stomach, Pancreatic tail
Superior Liver, adrenal gland
Spleen, adrenal gland

Fascial covering
Each kidney and suprarenal gland is enclosed within a common and layer of
investing fascia that is derived from the transversalis fascia into anterior and
posterior layers (Gerotas fascia).
Renal structure

Kidneys are surrounded by an outer cortex and an inner medulla which usually
contains between 6 and 10 pyramidal structures. The papilla marks the
innermost apex of these. They terminate at the renal pelvis, into the ureter.
Lying in a hollow within the kidney is the renal sinus. This contains:
1. Branches of the renal artery
2. Tributaries of the renal vein
3. Major and minor calyces's
4. Fat

Structures at the renal hilum


The renal vein lies most anteriorly, then renal artery (it is an end artery) and
the ureter lies most posterior.
Which of the following structures is not directly related to the right adrenal
gland?
A. Diaphragm posteriorly
B. Bare area of the liver anteriorly
C. Right renal vein
D. Inferior vena cava
E. Hepato-renal pouch

The right renal vein is very short and lies more inferiorly.
Adrenal gland anatomy
Anatomy

Location
Relationships of the
right adrenal
Relationships of the
left adrenal
Arterial supply
Venous drainage of
the right adrenal
Venous drainage of
the left adrenal

Superomedially to the upper pole of each kidney


Diaphragm-Posteriorly, Kidney-Inferiorly, Vena CavaMedially, Hepato-renal pouch and bare area of the liverAnteriorly
Crus of the diaphragm-Postero- medially, Pancreas and
splenic vessels-Inferiorly, Lesser sac and stomach-Anteriorly
Superior adrenal arteries- from inferior phrenic artery,
Middle adrenal arteries - from aorta, Inferior adrenal arteries
-from renal arteries
Via one central vein directly into the IVC
Via one central vein into the left renal vein

Mobilisation of the left lobe of the liver will facilitate surgical access to which of the
following?

A. Abdominal oesophagus
B. Duodenum
C. Right colic flexure
D. Right kidney
E. Pylorus of stomach
The fundus of the stomach is a posterior relation. The pylorus lies more
inferolaterally. During a total gastrectomy division of the ligaments holding the left
lobe of the liver will facilitate access to the proximal stomach and abdominal
oesophagus. This manoeuvre is seldom beneficial during a distal gastrectomy.
Liver
Structure of the liver
Right lobe
Supplied by right hepatic artery

Contains Couinard segments V to VIII (-/+Sg I)

Supplied by the left hepatic artery

Contains Couinard segments II to IV (+/- Sg1)

Part of the right lobe anatomically, functionally is part of the


left

Couinard segment IV

Porta hepatis lies behind

On the right lies the gallbladder fossa

On the left lies the fossa for the umbilical vein

Supplied by both right and left hepatic arteries

Couinard segment I

Lies behind the plane of the porta hepatis

Left lobe

Quadrate
lobe

Caudate lobe

Anterior and lateral to the inferior vena cava

Bile from the caudate lobe drains into both right and left
hepatic ducts

Detailed knowledge of Couinard segments is not required for MRCS Part A

Between the liver lobules are portal canals which contain the portal triad:
Hepatic Artery, Portal Vein, tributary of Bile Duct.

Relations of the liver


Anterior
Diaphragm
Xiphoid process

Postero inferiorly
Oesophagus
Stomach
Duodenum
Hepatic flexure of colon
Right kidney
Gallbladder
Inferior vena cava

Porta hepatis
Location Postero inferior surface, it joins nearly at right angles with the left sagittal
fossa, and separates the caudate lobe behind from the quadrate lobe in
front
Transmits
Common hepatic duct

Hepatic artery

Portal vein

Sympathetic and parasympathetic nerve fibres

Lymphatic drainage of the liver (and nodes)

Ligaments
Falciform ligament

2 layer fold peritoneum from the umbilicus to anterior


liver surface

Contains ligamentum teres (remnant umbilical vein)

On superior liver surface it splits into the coronary and


left triangular ligaments

Ligamentum teres Joins the left branch of the portal vein in the porta hepatis
Ligamentum
Remnant of ductus venosus
venosum
Arterial supply

Hepatic artery

Venous

Hepatic veins

Portal vein

Nervous supply

Sympathetic and parasympathetic trunks of coeliac plexus

A 32 year old man presents with an inguinal hernia and undergoes an


open surgical repair. The surgeons decide to place a mesh on the posterior
wall of the inguinal canal to complete the repair, which of the following
structures will lie posterior to the mesh?
A Transversalis fascia
.
B External oblique
.
C Rectus abdominis
.
D Obturator nerve
.
E None of the above
.
Inguinal canal walls: 'MALT: 2M, 2A, 2L, 2T':
Starting from superior, moving around in order to posterior:

Superior wall (roof): 2 Muscles:Internal oblique, transversus abdominis


Anterior wall: 2 Aponeuroses: Aponeurosis of external oblique,
Aponeurosis of internal oblique
Lower wall (floor): 2 Ligaments: Inguinal Ligament, Lacunar Ligament
Posterior wall: 2Ts: Transversalis fascia, Conjoint Tendon
This is actually quite a straightforward question. It is simply asking for the
structure that forms the posterior wall of the inguinal canal. This is
composed of the transversalis fascia, the conjoint tendon and more
laterally the deep inguinal ring.
Inguinal canal
Location

Above the inguinal ligament

The inguinal canal is 4cm long

Boundaries of the inguinal canal


Floor
External oblique aponeurosis

Inguinal ligament

Lacunar ligament

Internal oblique

Transversus abdominis

Roof

Anterior wall

External oblique aponeurosis

Posterior wall

Transversalis fascia

Conjoint tendon

Internal ring

Fibres of internal oblique

External ring

Laterally

Medially

Conjoint tendon

Contents
Males Spermatic cord and
ilioinguinal nerve

As it passes through the canal the


spermatic cord has 3 coverings:

External spermatic fascia from


external oblique aponeurosis

Cremasteric fascia

Internal spermatic fascia

Femal Round ligament of uterus


es
and ilioinguinal nerve
Related anatomy of the inguinal region
The boundaries of Hesselbachs triangle are commonly tested and
illustrated below:

Image sourced from Wikipedia

The image below demonstrates the close relationship of the vessels to the

lower limb with the inguinal canal. A fact to be borne in mind when
repairing hernial defects in this region.

Image sourced from Wikipedia

A 22 year old man is involved in a fight outside a nightclub. He is stabbed in the back,
on the left side, approximately 3cm below the 12th rib in the mid scapular line. The
structure most likely to be injured first as a result is the:
A. Spleen
B. Left kidney
C. Left adrenal gland
D. Left ureter
E. None of the above
The left kidney lies in this location and is the most likely structure to be injured. The
Spleen lies more superiorly, and the left adrenal and ureter are unlikely to be injured
in isolation.
Levels
Transpyloric plane
Level of the body of L1

Pylorus stomach

Left kidney hilum (L1- left one!)

Right hilum of the kidney (1.5cm lower than the left)

Fundus of the gallbladder

Neck of pancreas

Duodenojejunal flexure

Superior mesenteric artery

Portal vein

Left and right colic flexure

Root of the transverse mesocolon

2nd part of the duodenum

Upper part of conus medullaris

Spleen

Can be identified by asking the supine patient to sit up without using their arms. The
plane is located where the lateral border of the rectus muscle crosses the costal
margin.
Anatomical planes
Subcostal plane
Intercristal plane
Intertubercular plane

Lowest margin of 10th costal cartilage


Level of body L4 (highest point of iliac crest)
Level of body L5

Common level landmarks


Inferior mesenteric artery
L3
Bifurcation of aorta into common iliac arteries L4
Formation of IVC
L5 (union of common iliac veins)
Diaphragm apertures
Vena cava T8

Oesophagus T10

Aortic hiatus T12

A 23 year old man is undergoing an hernia repair and the mesh is to be


sutured to the inguinal ligament. From which of the following does the
inguinal ligament arise?
A Transversus abdominis fascia
.
B Internal oblique
.
C Rectus sheath
.
D Rectus abdominis muscle
.
E. External oblique aponeurosis

The inguinal ligament is formed by the external oblique aponeurosis. It


runs from the pubic tubercle to the anterior superior iliac spine.
Abdominal wall
The 2 main muscles of the abdominal wall are the rectus abdominis
(anterior) and the quadratus lumborum (posterior).
The remaining abdominal wall consists of 3 muscular layers. Each muscle
passes from the lateral aspect of the quadratus lumborum posteriorly to
the lateral margin of the rectus sheath anteriorly. Each layer is muscular
posterolaterally and aponeurotic anteriorly.

Image sourced from Wikipedia

Muscles of abdominal wall


External
Lies most superficially
oblique

Internal
oblique

Transversu
s
abdominis

Surgical notes

Originates from 5th to 12th ribs

Inserts into the anterior half of the outer aspect of the


iliac crest, linea alba and pubic tubercle

More medially and superiorly to the arcuate line, the


aponeurotic layer overlaps the rectus abdominis
muscle

The lower border forms the inguinal ligament

The triangular expansion of the medial end of the


inguinal ligament is the lacunar ligament.

Arises from the thoracolumbar fascia, the anterior 2/3


of the iliac crest and the lateral 2/3 of the inguinal
ligament

The muscle sweeps upwards to insert into the


cartilages of the lower 3 ribs

The lower fibres form an aponeurosis that runs from


the tenth costal cartilage to the body of the pubis

At its lowermost aspect it joins the fibres of the


aponeurosis of transversus abdominis to form the
conjoint tendon.

Innermost muscle

Arises from the inner aspect of the costal cartilages of


the lower 6 ribs , from the anterior 2/3 of the iliac crest
and lateral 1/3 of the inguinal ligament

Its fibres run horizontally around the abdominal wall


ending in an aponeurosis. The upper part runs
posterior to the rectus abdominis. Lower down the
fibres run anteriorly only.

The rectus abdominis lies medially running from the


pubic crest and symphysis to insert into the xiphoid
process and 5th, 6th and 7th costal cartilages. The
muscles lies in a aponeurosis as described above.

Nerve supply: anterior primary rami of T7-12

During abdominal surgery it is usually necessary to divide either the


muscles or their aponeuroses. During a midline laparotomy it is desirable
to divide the aponeurosis. This will leave the rectus sheath intact above
the arcuate line and the muscles intact below it. Straying off the midline
will often lead to damage to the rectus muscles, particularly below the
arcuate line where they may often be in close proximity to each other.

A 56 year old man is undergoing a high anterior resection. Which of the following
structures is at greatest risk of injury in this procedure?
A. Superior mesenteric artery
B. Left ureter
C. External iliac vein
D. External iliac artery
E. Inferior vena cava
A careless surgeon may damage all of these structures. However, the structure at
greatest risk and most frequently encountered is the left ureter.
Colon anatomy
The colon is about 1.5m long although this can vary considerably.
Components:

Ascending colon

Transverse colon

Descending colon

Sigmoid colon

Arterial supply
Superior mesenteric artery and inferior mesenteric artery: linked by the marginal
artery.
Ascending colon: ileocolic and right colic arteries
Transverse colon: middle colic artery
Descending and sigmoid colon: left colic artery
Venous drainage
From regional veins (that accompany arteries) to superior and inferior mesenteric vein
Lymphatic drainage
Initially along nodal chains that accompany supplying arteries, then para-aortic nodes.

Embryology
Midgut- Second part of duodenum to 2/3 transverse colon
Hindgut- Distal 1/3 transverse colon to anus
Peritoneal location
The right and left colon are part intraperitoneal and part extraperitoneal. The sigmoid
and transverse colon are generally wholly intraperitoneal. This has implications for
the sequelae of perforations, which will tend to result in generalised peritonitis in the
wholly intra peritoneal segments.
A 42 year old lady undergoes a difficult cholecystectomy and significant bleeding is
occurring. The surgeons place a vascular clamp transversely across the anterior border
of the epiploic foramen. Which of the following structures will be occluded in this
manoeuvre?
A. Cystic artery
B. Cystic duct
C. Left gastric artery
D. Portal vein
E. None of the above
The portal vein, hepatic artery and common bile duct are occluded.
Epiploic Foramen
The epiploic foramen has the following boundaries:
Anteriorly (in the free edge of the Bile duct to the right, portal vein behind and
lesser omentum)
hepatic artery to the left.
Posteriorly
Inferior vena cava
Inferiorly
1st part of the duodenum
Superiorly
Caudate process of the liver
During liver surgery bleeding may be controlled using a Pringles manoeuvre, this
involves placing a vascular clamp across the anterior aspect of the epiploic foramen.
Thereby occluding:

Common bile duct

Hepatic artery

Portal vein

Rate question: 1

2
3
4
5

A 56 year old lady undergoes a Hartmans style resection of the sigmoid colon, with
ligation of the vessels close to the colon. Which of the following vessels will be
responsible to supplying the rectal stump directly?
A. Superior mesenteric artery
B. Middle colic artery
C. Superior rectal artery
D. Inferior mesenteric artery
E. External iliac artery
This question is addressing the blood supply to the rectum. Which is supplied by the
superior rectal artery. High ligation of the IMA may compromise this structure.
However, the question states that during the Hartmans procedure the vessels were
ligated close to the bowel. Implying that the superior rectal was preserved.
Rectum
The rectum is approximately 12 cm long. It is a capacitance organ. It has both intra
and extraperitoneal components. The transition between the sigmoid colon is marked
by the disappearance of the tenia coli.The extra peritoneal rectum is surrounded by
mesorectal fat that also contains lymph nodes. This mesorectal fatty layer is removed
surgically during rectal cancer surgery (Total Mesorectal Excision). The fascial layers
that surround the rectum are important clinical landmarks, anteriorly lies the fascia of
Denonvilliers. Posteriorly lies Waldeyers fascia.
Extra peritoneal rectum

Posterior upper third

Posterior and lateral middle third

Whole lower third

Relations
Anteriorly (Males)

Rectovesical pouch
Bladder
Prostate
Seminal vesicles

Anteriorly (Females)

Recto-uterine pouch (Douglas)


Cervix
Vaginal wall
Sacrum
Coccyx
Middle sacral artery
Levator ani
Coccygeus

Posteriorly
Laterally
Arterial supply
Superior rectal artery
Venous drainage
Superior rectal vein
Lymphatic drainage

Mesorectal lymph nodes (superior to dentate line)

Internal iliac and then para-aortic nodes

Inguinal nodes (inferior to dentate line)

On inspecting the caecum, which of the following structures is most likely to be


identified at the point at which all the tenia coli converge?
A. Gonadal vessels
B. Appendix base
C. Appendix tip
D. Ileocaecal valve
E. Ileocolic artery
The tenia coli converge at the base of the appendix.
Caecum
Location

Proximal right colon below the ileocaecal valve

Intraperitoneal

Psoas

Posterior relations

Iliacus

Femoral nerve

Genitofemoral nerve

Gonadal vessels

Anterior relations
Greater omentum
Arterial supply
Ileocolic artery
Lymphatic drainage Mesenteric nodes accompany the venous drainage

The caecum is the most distensible part of the colon and in complete large
bowel obstruction with a competent ileocaecal valve the most likely site of
eventual perforation.

Which of the following structures lies most posteriorly at the porta hepatis?
A. Cystic artery
B. Common hepatic artery
C. Left hepatic artery
D. Portal vein
E. Common bile duct
The portal vein is the most posterior structure at the porta hepatis.The common bile
duct is a continuation of the common hepatic duct and is formed by the union of the
common hepatic duct and the cystic duct.
Liver
Structure of the liver
Right lobe
Supplied by right hepatic artery

Contains Couinard segments V to VIII (-/+Sg I)

Supplied by the left hepatic artery

Contains Couinard segments II to IV (+/- Sg1)

Left lobe

Quadrate
lobe

Part of the right lobe anatomically, functionally is part of the


left

Couinard segment IV

Porta hepatis lies behind

On the right lies the gallbladder fossa

On the left lies the fossa for the umbilical vein

Supplied by both right and left hepatic arteries

Couinard segment I

Lies behind the plane of the porta hepatis

Anterior and lateral to the inferior vena cava

Bile from the caudate lobe drains into both right and left
hepatic ducts

Caudate lobe

Detailed knowledge of Couinard segments is not required for MRCS Part A

Between the liver lobules are portal canals which contain the portal triad:
Hepatic Artery, Portal Vein, tributary of Bile Duct.

Relations of the liver


Anterior
Diaphragm
Xiphoid process

Postero inferiorly
Oesophagus
Stomach
Duodenum
Hepatic flexure of colon
Right kidney
Gallbladder

Inferior vena cava


Porta hepatis
Location Postero inferior surface, it joins nearly at right angles with the left sagittal
fossa, and separates the caudate lobe behind from the quadrate lobe in
front
Transmits
Common hepatic duct

Hepatic artery

Portal vein

Sympathetic and parasympathetic nerve fibres

Lymphatic drainage of the liver (and nodes)

Ligaments
Falciform ligament

2 layer fold peritoneum from the umbilicus to anterior


liver surface

Contains ligamentum teres (remnant umbilical vein)

On superior liver surface it splits into the coronary and


left triangular ligaments

Ligamentum teres Joins the left branch of the portal vein in the porta hepatis
Ligamentum
Remnant of ductus venosus
venosum
Arterial supply

Hepatic artery

Venous

Hepatic veins

Portal vein

Nervous supply

Sympathetic and parasympathetic trunks of coeliac plexus

A 76 year old man is undergoing an abdominal aortic aneurysm


repair. The surgeons occlude the aorta with two clamps, the inferior
clamp being placed at the point of aortic bifurcation. Which of the
following vertebral bodies will lie posterior to the clamp at this
level?
A L1
.
B T10
.
C L4
.
D L5
.
E. L2

The aorta bifurcates at L4. An important landmark that is tested


frequently.
Abdominal aorta
Abdominal aortic topography

Origin

T12

Termination

L4

Posterior relations

L1-L4 Vertebral bodies

Anterior relations

Lesser omentum
Liver
Left renal vein
Inferior mesenteric vein
Third part of duodenum
Pancreas
Parietal peritoneum
Peritoneal cavity

Right lateral relations

Right crus of the diaphragm


Cisterna chyli
Azygos vein
IVC (becomes posterior distally)

Left lateral relations

4th part of duodenum


Duodenal-jejunal flexure
Left sympathetic trunk

The abdominal aorta

Image sourced from Wikipedia

Which of the following statements relating to the greater omentum is false?


A. It is less well developed in children under 5.
B. It has no relationship to the lesser sac.
C. It contains the gastroepiploic arteries.
D. Has an attachment to the transverse colon.
E. It may be a site of metastatic disease in ovarian cancer.
It is connected with the lesser sac and the transverse colon. This plane is entered when
performing a colonic resection. It is a common site of metastasis in many visceral
malignancies.
Omentum

The omentum is divided into two parts which invest the stomach. Giving rise
to the greater and lesser omentum. The greater omentum is attached to the
inferolateral border of the stomach and houses the gastro-epiploic arteries.

It is of variable size but is less well developed in children. This is important as


the omentum confers protection against visceral perforation (e.g.
Appendicitis).

Inferiorly between the omentum and transverse colon is one potential entry
point into the lesser sac.

Several malignant processes may involve the omentum of which ovarian


cancer is the most notable.

A 48 year old man with newly diagnosed hypertension is found to have a


phaeochromocytoma of the left adrenal gland and is due to undergo a
laparoscopic left adrenalectomy. Which of the following structures is not
directly related to the left adrenal gland?
A. Crus of the diaphragm
B. Lesser curvature of the stomach
C. Splenic hilum
D. Pancreas
E. Splenic artery

The splenic hilum lies more laterally and is therefore not a direct relation of
the left adrenal gland.
Adrenal gland anatomy
Anatomy

Location
Relationships of the
right adrenal
Relationships of the
left adrenal
Arterial supply
Venous drainage of
the right adrenal
Venous drainage of
the left adrenal

Superomedially to the upper pole of each kidney


Diaphragm-Posteriorly, Kidney-Inferiorly, Vena CavaMedially, Hepato-renal pouch and bare area of the liverAnteriorly
Crus of the diaphragm-Postero- medially, Pancreas and
splenic vessels-Inferiorly, Lesser sac and stomach-Anteriorly
Superior adrenal arteries- from inferior phrenic artery,
Middle adrenal arteries - from aorta, Inferior adrenal arteries
-from renal arteries
Via one central vein directly into the IVC
Via one central vein into the left renal vein

1
2
Rate question: 3
4
5

An 18 year old boy is undergoing an appendicectomy for appendicitis. At which of


the following locations is the appendix most likely to be found?
A. Pre ileal
B. Pelvic
C. Retrocaecal
D. Post ileal
E. None of the above
Most appendixes lie in the retrocaecal position. If a retrocaecal appendix is difficult to
remove then mobilisation of the right colon significantly improves access.
Appendix

Location: Base of caecum.

Up to 10cm long.

Mainly lymphoid tissue (Hence mesenteric adenitis may mimic appendicitis).

Caecal taenia coli converge at base of appendix and form a longitudinal


muscle cover over the appendix. This convergence should facilitate its
identification at surgery if it is retrocaecal and difficult to find (which it can be
when people start doing appendicectomies!)

Arterial supply: Appendicular artery (branch of the ileocolic).

It is intra peritoneal.

McBurney's point

1/3 of the way along a line drawn from the Anterior Superior Iliac Spine to the
Umbilicus

6 Positions:

Retrocaecal 74%

Pelvic 21%

Postileal

Subcaecal

Paracaecal

Preileal

A 56 year old man is undergoing a pancreatectomy for carcinoma. During resection of


the gland which of the following structures will the surgeon not encounter posterior to
the pancreas itself?
A. Left crus of the diaphragm
B. Superior mesenteric vein
C. Common bile duct
D. Portal vein
E. Gastroduodenal artery
Theme from 2010 Exam
The gastroduodenal artery lies anterior to the pancreas.
Pancreas
The pancreas is a retroperitoneal organ and lies posterior to the stomach. It may be
accessed surgically by dividing the peritoneal reflection that connects the greater
omentum to the transverse colon. The pancreatic head sits in the curvature of the
duodenum. It's tail lies close to the hilum of the spleen, a site of potential injury
during splenectomy.
Relations
Posterior to the pancreas
Pancreatic head
Inferior vena cava
Common bile duct
Right and left renal veins

Pancreatic neck
Pancreatic body-

Pancreatic tail
Anterior to the pancreas
Pancreatic head

Pancreatic body
Pancreatic tail

Superior mesenteric vein and artery


Superior mesenteric vein, portal vein
Left renal vein
Crus of diaphragm
Psoas muscle
Adrenal gland
Kidney
Aorta
Left kidney
1st part of the duodenum
Pylorus
Gastroduodenal artery
SMA and SMV(uncinate process)
Stomach
Duodenojejunal flexure
Splenic hilum

Superior to the pancreas


Coeliac trunk and its branches common hepatic artery and splenic artery
Grooves of the head of the pancreas
2nd and 3rd part of the duodenum
Arterial supply

Head: pancreaticoduodenal artery

Rest: splenic artery

Venous drainage

Head: superior mesenteric vein

Body and tail: splenic vein

Ampulla of Vater

Merge of pancreatic duct and common bile duct

Is an important landmark, halfway along the second part of the duodenum, that
marks the anatomical transition from foregut to midgut (also the site of
transition between regions supplied by coeliac trunk and SMA).

A 55 year old man is admitted with a brisk haematemesis. He is taken to the


endoscopy department and an upper GI endoscopy is performed by the
gastroenterologist. He identifies an ulcer on the posterior duodenal wall and spends an
eternity trying to control the bleeding with all the latest haemostatic techniques. He
eventually asks the surgeons for help. A laparotomy and anterior duodenotomy are
performed, as the surgeon opens the duodenum a vessel is spurting blood into the
duodenal lumen. From which of the following does this vessel arise?
A. Left gastric artery
B. Common hepatic artery
C. Right hepatic artery
D. Superior mesenteric artery
E. Splenic artery
The vessel will be the gastroduodenal artery, this arises from the common hepatic
artery.
Gastroduodenal artery
Supplies
Pylorus, proximal part of the duodenum, and indirectly to the pancreatic head (via the
anterior and posterior superior pancreaticoduodenal arteries)
Path
Most commonly arises from the common hepatic artery of the coeliac trunk
Terminates by bifurcating into the right gastroepiploic artery and the superior

pancreaticoduodenal artery
Image showing stomach reflected superiorly to illustrate the relationship of the
gastroduodenal artery to the first part of the duodenum

Which of the following is not a content of the rectus sheath?


A Pyramidalis
.
B Superior epigastric artery
.
C Inferior epigastric vein
.
D Internal iliac artery
.
E. Rectus abdominis

The rectus sheath also contains:

superior epigastric vein


inferior epigastric artery
Abdominal wall
The 2 main muscles of the abdominal wall are the rectus abdominis
(anterior) and the quadratus lumborum (posterior).
The remaining abdominal wall consists of 3 muscular layers. Each muscle
passes from the lateral aspect of the quadratus lumborum posteriorly to
the lateral margin of the rectus sheath anteriorly. Each layer is muscular
posterolaterally and aponeurotic anteriorly.

Image sourced from Wikipedia

Muscles of abdominal wall


External
Lies most superficially
oblique

Internal
oblique

Originates from 5th to 12th ribs

Inserts into the anterior half of the outer aspect of the


iliac crest, linea alba and pubic tubercle

More medially and superiorly to the arcuate line, the


aponeurotic layer overlaps the rectus abdominis
muscle

The lower border forms the inguinal ligament

The triangular expansion of the medial end of the


inguinal ligament is the lacunar ligament.

Arises from the thoracolumbar fascia, the anterior 2/3


of the iliac crest and the lateral 2/3 of the inguinal
ligament

The muscle sweeps upwards to insert into the


cartilages of the lower 3 ribs

The lower fibres form an aponeurosis that runs from

the tenth costal cartilage to the body of the pubis

Transversu
s
abdominis

At its lowermost aspect it joins the fibres of the


aponeurosis of transversus abdominis to form the
conjoint tendon.

Innermost muscle

Arises from the inner aspect of the costal cartilages of


the lower 6 ribs , from the anterior 2/3 of the iliac crest
and lateral 1/3 of the inguinal ligament

Its fibres run horizontally around the abdominal wall


ending in an aponeurosis. The upper part runs
posterior to the rectus abdominis. Lower down the
fibres run anteriorly only.

The rectus abdominis lies medially running from the


pubic crest and symphysis to insert into the xiphoid
process and 5th, 6th and 7th costal cartilages. The
muscles lies in a aponeurosis as described above.

Nerve supply: anterior primary rami of T7-12

Surgical notes
During abdominal surgery it is usually necessary to divide either the
muscles or their aponeuroses. During a midline laparotomy it is desirable
to divide the aponeurosis. This will leave the rectus sheath intact above
the arcuate line and the muscles intact below it. Straying off the midline
will often lead to damage to the rectus muscles, particularly below the
arcuate line where they may often be in close proximity to each other.
Which of the following vessels does not drain directly into the inferior vena
cava?
A Superior mesenteric vein
.
B Right common iliac
.
C Right hepatic vein
.

D Left hepatic vein


.
E. Right testicular vein

The superior mesenteric vein drains into the portal vein. The right and left
hepatic veins drain into it directly, this can account for major bleeding in
more extensive liver shearing type injuries.
Inferior vena cava
Origin

L5

Path

Left and right common iliac veins merge to form the IVC.

Passes right of midline

Paired segmental lumbar veins drain into the IVC throughout its
length

The right gonadal vein empties directly into the cava and the left
gonadal vein generally empties into the left renal vein.

The next major veins are the renal veins and the hepatic veins

Pierces the central tendon of diaphragm at T8

Right atrium

Image sourced from Wikipedia

Relations
Anterior Small bowel, first and third part of duodenum, head of pancreas,
ly
liver and bile duct, right common iliac artery, right gonadal
artery
Posteri Right renal artery, right psoas, right sympathetic chain, coeliac
orly
ganglion
Levels
Level Vein
T8

Hepatic vein, inferior phrenic vein, pierces diaphragm

L1

Suprarenal veins, renal vein

L2

Gonadal vein

L1-5

Lumbar veins

L5

Common iliac vein, formation of IVC

A 17 year old male has a suspected testicular torsion and the scrotum is to be explored
surgically. The surgeon incises the skin and then the dartos muscle. What is the next
tissue layer that will be encountered during the dissection?
A. Visceral layer of the tunica vaginalis

B. Cremasteric fascia
C. Parietal layer of the tunica vaginalis
D. External spermatic fascia
E. Internal spermatic fascia
The layers that will be encountered are (in order):
1. Skin
2. Dartos fascia and muscle
3. External spermatic fascia
4. Cremasteric muscle and fascia
5. Internal spermatic fascia
6. Parietal layer of the tunica vaginalis
The layers of the spermatic cord and scrotum are a popular topic in the MRCS exam.
Scrotal and testicular anatomy
Spermatic cord
Formed by the vas deferens and is covered by the following structures:
Layer
Origin
Internal spermatic fascia
Transversalis fascia
Cremasteric fascia
From the fascial coverings of internal oblique
External spermatic fascia
External oblique aponeurosis
Contents of the cord
Vas deferens
Testicular artery
Artery of vas deferens
Cremasteric artery
Pampiniform plexus
Sympathetic nerve fibres
Genital branch of the
genitofemoral nerve
Lymphatic vessels

Transmits sperm and accessory gland


secretions
Branch of abdominal aorta supplies testis and
epididymis
Arises from inferior vesical artery
Arises from inferior epigastic artery
Venous plexus, drains into right or left
testicular vein
Lie on arteries, the parasympathetic fibres lie
on the vas
Supplies cremaster
Drain to lumbar and para-aortic nodes

Scrotum

Composed of skin and closely attached dartos fascia.

Arterial supply from the anterior and posterior scrotal arteries

Lymphatic drainage to the inguinal lymph nodes

Parietal layer of the tunica vaginalis is the innermost layer

Testes

The testes are surrounded by the tunica vaginalis (closed peritoneal sac). The
parietal layer of the tunica vaginalis adjacent to the internal spermatic fascia.

The testicular arteries arise from the aorta immediately inferiorly to the renal
arteries.

The pampiniform plexus drains into the testicular veins, the left drains into the
left renal vein and the right into the inferior vena cava.

Lymphatic drainage is to the para-aortic nodes.

A 25 year old man is stabbed in the groin and the area, which lies within
the femoral triangle is explored. Which structure forms the lateral wall of
the femoral triangle?
A Adductor longus
.
B Pectineus
.
C Adductor magnus
.
D Sartorius
.
E. Conjoint tendon

The sartorius forms the lateral wall of the femoral triangle (see below).
Femoral triangle anatomy
Boundaries
Superio Inguinal ligament
rly

Laterall Sartorius
y
Mediall Adductor longus
y
Floor

Iliopsoas, adductor longus and pectineus

Roof

Fascia lata and Superficial fascia

Superficial inguinal lymph nodes (palpable below the


inguinal ligament)

Great saphenous vein

Image sourced from Wikipedia

Contents

Femoral vein (medial to lateral)

Femoral artery-pulse palpated at the mid inguinal point

Femoral nerve

Deep and superficial inguinal lymph nodes

Lateral cutaneous nerve

Great saphenous vein

Femoral branch of the genitofemoral nerve

A 19 year old man undergoes an open inguinal hernia repair. The cord is
mobilised and the deep inguinal ring identified. Which of the following
structures forms its lateral wall?
A External oblique aponeurosis
.
B Transversalis fascia
.
C Conjoint tendon
.
D Inferior epigastric artery
.
E. Inferior epigastric vein

The transversalis fascia forms the superolateral edge of the deep inguinal
ring. The epigastric vessels form its inferomedial wall.
Inguinal canal
Location

Above the inguinal ligament

The inguinal canal is 4cm long

Boundaries of the inguinal canal


Floor
External oblique aponeurosis

Inguinal ligament

Lacunar ligament

Internal oblique

Transversus abdominis

Roof

Anterior wall

External oblique aponeurosis

Posterior wall

Transversalis fascia

Conjoint tendon

Internal ring

Fibres of internal oblique

External ring

Conjoint tendon

Laterally

Medially

Contents
Males Spermatic cord and
ilioinguinal nerve

As it passes through the canal the


spermatic cord has 3 coverings:

External spermatic fascia from


external oblique aponeurosis

Cremasteric fascia

Internal spermatic fascia

Femal Round ligament of uterus


es
and ilioinguinal nerve
Related anatomy of the inguinal region
The boundaries of Hesselbachs triangle are commonly tested and
illustrated below:

Image sourced from Wikipedia

The image below demonstrates the close relationship of the vessels to the
lower limb with the inguinal canal. A fact to be borne in mind when
repairing hernial defects in this region.

Image sourced from Wikipedia

A 34 year old lady presents with symptoms of faecal incontinence. Ten years
previously she gave birth to a child by normal vaginal delivery. Injury to which of the
following nerves is most likely to account for this process?
A. Genitofemoral
B. Ilioinguinal
C. Pudendal
D. Hypogastric autonomic nerve
E. Obturator
Theme from April 2012 Exam
Damage to the pudendal nerve is classically associated with faecal incontinence and it
is for this reason that sacral neuromodulation is a popular treatment for the condition.
Injury to the hypogastric autonomic nerves is an aetiological factor in the
development of constipation.
Pudendal nerve
The pudendal nerve arises from nerve roots S2, S3 and S4 and exits the pelvis through
the greater sciatic foramen. It re-enters the pelvis through the lesser sciatic foramen. It
travels inferior to give innervation to the anal sphincters and external urethral
sphincter. It also provides cutaneous innervation to the region of perineum
surrounding the anus and posterior vulva.
Traction and compression of the pudendal nerve by the foetus in late pregnancy may
result in late onset pudendal neuropathy which may be part of the process involved in
the development of faecal incontinence.
A 56 year old man undergoes an abdomino-perineal excision of the rectum. He is
assessed in the outpatient clinic post operatively. His wounds are well healed.
However, he complains of impotence. Which of the following best explains this
problem?
A. Sciatic nerve injury
B. Damage to the internal iliac artery
C. Damage to the hypogastric nerve plexus
D. Damage to the vas
E. Damage to the genitofemoral nerve
Theme from 2012 Exam
Autonomic supply to the penis is via the hypogastric plexus of nerves. These may be
damaged during mobilisation of the proximal rectum from the sacrum and result in

impotence post operatively. The addition of radiotherapy greatly increases the risks of
impotence following surgery.
Penile erection
Physiology of erection
Autonomic
Sympathetic nerves originate from T11-L2 and parasympathetic
nerves from S2-4 join to form pelvic plexus.

Somatic
nerves

Parasympathetic discharge causes erection, sympathetic


discharge causes ejaculation and detumescence.

Supplied by dorsal penile and pudendal nerves. Efferent signals are


relayed from Onufs nucleus (S2-4) to innervate ischiocavernosus and
bulbocavernosus muscles.

Autonomic discharge to the penis will trigger the veno-occlusive mechanism which
triggers the flow of arterial blood into the penile sinusoidal spaces. As the inflow
increases the increased volume in this space will secondarily lead to compression of
the subtunical venous plexus with reduced venous return. During the detumesence
phase the arteriolar constriction will reduce arterial inflow and thereby allow venous
return to normalise.
Priapism
Prolonged unwanted erection, in the absence of sexual desire, lasting more than 4
hours.
Classification of priaprism
Low flow
Due to veno-occlusion (high intracavernosal pressures).
priaprism
Most common type

High flow
priaprism

Often painful

Often low cavernosal flow

If present for >4 hours requires emergency treatment

Due to unregulated arterial blood flow.

Recurrent
priaprism
Causes

Usually presents as semi rigid painless erection

Typically seen in sickle cell disease, most commonly of high


flow type.

Intracavernosal drug therapies (e.g. for erectile dysfunction>

Blood disorders such as leukaemia and sickle cell disease

Neurogenic disorders such as spinal cord transection

Trauma to penis resulting in arterio-venous malformations

Tests

Exclude sickle cell/ leukaemia

Consider blood sampling from cavernosa to determine whether high or low


flow (low flow is often hypoxic)

Management

Ice packs/ cold showers

If due to low flow then blood may be aspirated from copora or try
intracavernosal alpha adrenergic agonists.

Delayed therapy of low flow priaprism may result in erectile dysfunction.

Which of the following is not a branch of the descending abdominal aorta?


A. Inferior mesenteric artery
B. Inferior phrenic artery
C. Superior mesenteric artery
D. Superior phrenic artery
E. Renal artery

Mnemonic for the Descending abdominal aorta branches from diaphragm to iliacs:
'Prostitutes Cause Sagging Swollen Red Testicles [in men] Living In Sin':
Phrenic [inferior]
Celiac
Superior mesenteric
Suprarenal [middle]
Renal
Testicular ['in men' only]
Lumbars

Inferior mesenteric
Sacral

The superior phrenic artery branches from the aorta in the thorax.
Abdominal aortic branches

Branches
Inferior phrenic
Coeliac
Superior mesenteric
Middle suprarenal
Renal
Gonadal
Lumbar
Inferior mesenteric
Median sacral
Common iliac

Level
T12 (Upper border)
T12
L1
L1
L1-L2
L2
L1-L4
L3
L4
L4

Paired
Yes
No
No
Yes
Yes
Yes
Yes
No
No
Yes

Type
Parietal
Visceral
Visceral
Visceral
Visceral
Visceral
Parietal
Visceral
Parietal
Terminal

A 23 year old man is admitted with a suspected ureteric colic. A KUB style x-ray is
obtained. In which of the following locations is the stone most likely to be visualised?
A. The tips of the transverse processes between L2 and L5
B. The tips of transverse processes between T10-L1
C. At the crest of the ilium
D. Over the S3 foramina
E. Over the sacrococcygeal joint
The ureter lies anterior to L2 to L5 and stones may be visualised at these points, they
may also be identified over the sacro-iliac joints.
Ureter

25-35 cm long

Muscular tube lined by transitional epithelium

Surrounded by thick muscular coat. Becomes 3 muscular layers as it crosses


the bony pelvis

Retroperitoneal structure overlying transverse processes L2-L5

Lies anterior to bifurcation of iliac vessels

Blood supply is segmental; renal artery, aortic branches, gonadal branches,


common iliac and internal iliac

Lies beneath the uterine artery

A 55 year old man is due to undergo a radical prostatectomy for


carcinoma of the prostate gland. Which of the following vessels directly
supplies the prostate?
A External iliac artery
.
B Common iliac artery
.
C Internal iliac artery
.
D Inferior vesical artery
.
E. None of the above

The arterial supply to the prostate gland is from the inferior vesical artery,
it is a branch of the prostatovesical artery. The prostatovesical artery
usually arises from the internal pudendal and inferior gluteal arterial
branches of the internal iliac artery.
Prostate gland
The prostate gland is approximately the shape and size of a walnut and is
located inferior to the bladder. It is separated from the rectum by
Denonvilliers fascia and its blood supply is derived from the internal iliac
vessels. The internal sphincter lies at the apex of the gland and may be
damaged during prostatic surgery, affected individuals may complain of
retrograde ejaculation.
Summary of prostate gland
Arterial

Inferior vesical artery (from internal iliac)

supply
Venous
drainage

Prostatic venous plexus (to paravertebral veins)

Lymphatic
drainage

Internal iliac nodes

Innervation Inferior hypogastric plexus


Dimensions

Transverse diameter (4cm)

AP diameter (2cm)

Height (3cm)

Posterior lobe: posterior to urethra

Median lobe: posterior to urethra, in between


ejaculatory ducts

Lateral lobes x 2

Isthmus

Peripheral zone: subcapsular portion of posterior


prostate. Most prostate cancers are here

Central zone

Transition zone

Stroma

Lobes

Zones

Relations
Anterior Pubic symphysis
Prostatic venous plexus
Posterior Denonvilliers fascia
Rectum
Ejaculatory ducts
Lateral

Venous plexus (lies on prostate)


Levator ani (immediately below the puboprostatic ligaments)

Image sourced from Wikipedia


From which of the following embryological structures is the ureter derived?
A. Uranchus
B. Wolffian duct
C. Vitello-intestinal duct
D. Mesonephric duct
E. Cloaca
The ureter develops from the mesonephric duct.
Ureter

25-35 cm long

Muscular tube lined by transitional epithelium

Surrounded by thick muscular coat. Becomes 3 muscular layers as it crosses


the bony pelvis

Retroperitoneal structure overlying transverse processes L2-L5

Lies anterior to bifurcation of iliac vessels

Blood supply is segmental; renal artery, aortic branches, gonadal branches,


common iliac and internal iliac

Lies beneath the uterine artery

A 56 year old man is having a long venous line inserted via the femoral
vein into the right atrium for CVP measurements. The catheter is
advanced through the IVC. At which of the following levels does this vessel
enter the thorax?
A L2
.
B T10
.
C L1
.
D T8
.
E. T6

Theme from 2010 Exam


Theme from September 2012 Exam
The IVC passes through the diaphragm at T8.
Inferior vena cava
Origin

L5

Path

Left and right common iliac veins merge to form the IVC.

Passes right of midline

Paired segmental lumbar veins drain into the IVC throughout its
length

The right gonadal vein empties directly into the cava and the left
gonadal vein generally empties into the left renal vein.

The next major veins are the renal veins and the hepatic veins

Pierces the central tendon of diaphragm at T8

Right atrium

Image sourced from Wikipedia

Relations
Anterior Small bowel, first and third part of duodenum, head of pancreas,
ly
liver and bile duct, right common iliac artery, right gonadal
artery
Posteri Right renal artery, right psoas, right sympathetic chain, coeliac
orly
ganglion
Levels
Level Vein
T8

Hepatic vein, inferior phrenic vein, pierces diaphragm

L1

Suprarenal veins, renal vein

L2

Gonadal vein

L1-5

Lumbar veins

L5

Common iliac vein, formation of IVC

A 62 year old man is undergoing a left hemicolectomy for carcinoma of the


descending colon. The registrar commences mobilisation of the left colon by pulling
downwards and medially. Blood soon appears in the left paracolic gutter. The most
likely source of bleeding is the:
A. Marginal artery
B. Left testicular artery
C. Spleen
D. Left renal vein
E. None of the above
The spleen is commonly torn by traction injuries in colonic surgery. The other
structures are associated with bleeding during colonic surgery but would not manifest
themselves as blood in the paracolic gutter prior to incision of the paracolonic
peritoneal edge.
Left colon
Position

As the left colon passes inferiorly its posterior aspect becomes extraperitoneal,
and the ureter and gonadal vessels are close posterior relations that may
become involved in disease processes

At a level of L3-4 (variable) the left colon becomes the sigmoid colon and
wholly intraperitoneal once again

The sigmoid colon is a highly mobile structure and may even lie of the right
side of the abdomen

It passes towards the midline, the taenia blend and this marks the transition
between sigmoid colon and upper rectum.

Blood supply

Inferior mesenteric artery

However, the marginal artery (from the right colon) contributes and this
contribution becomes clinically significant when the IMA is divided surgically
(e.g. During AAA repair)

52 year old female renal patient needs a femoral catheter to allow for
haemodialysis. Which of the structures listed below is least likely to be
encountered during its insertion?
A Great saphenous vein
.
B Deep circumflex iliac artery
.
C Superficial circumflex iliac artery
.
D Femoral vein
.
E. Femoral branch of the genitofemoral nerve

Femoral access catheters are typically inserted in the region of the femoral
triangle. Therefore the physician may encounter the femoral, vein, nerve,
branches of the femoral artery and tributaries of the femoral vein. The
deep circumflex iliac artery arises above the inguinal ligament and is
therefore less likely to be encountered than the superficial circumflex iliac
artery which arises below the inguinal ligament.
Femoral triangle anatomy
Boundaries
Superio Inguinal ligament
rly
Laterall Sartorius
y
Mediall Adductor longus
y
Floor

Iliopsoas, adductor longus and pectineus

Roof

Fascia lata and Superficial fascia

Superficial inguinal lymph nodes (palpable below the

inguinal ligament)

Great saphenous vein

Image sourced from Wikipedia

Contents

Femoral vein (medial to lateral)

Femoral artery-pulse palpated at the mid inguinal point

Femoral nerve

Deep and superficial inguinal lymph nodes

Lateral cutaneous nerve

Great saphenous vein

Femoral branch of the genitofemoral nerve

A 53 year old man with a chronically infected right kidney is due to undergo a
nephrectomy. Which of the following structures would be encountered first during a
posterior approach to the hilum of the right kidney?
A. Right renal artery
B. Ureter
C. Right renal vein
D. Inferior vena cava
E. Right testicular vein
The ureter is the most posterior structure at the hilum of the right kidney and would
therefore be encountered first during a posterior approach.
Renal arteries

The right renal artery is longer than the left renal artery

The renal vein/artery/pelvis enter the kidney at the hilum

Relations

Right:

Anterior- IVC, right renal vein, the head of the pancreas, and the descending part of
the duodenum.

Left:

Anterior- left renal vein, the tail of the pancreas.


Branches

The renal arteries are direct branches off the aorta (upper border of L2)

In 30% there may be accessory arteries (mainly left side). Instead of entering
the kidney at the hilum, they usually pierce the upper or lower part of the
organ.

Before reaching the hilum of the kidney, each artery divides into four or five
segmental branches (renal vein anterior and ureter posterior); which then
divide within the sinus into lobar arteries supplying each pyramid and cortex.

Each vessel gives off some small inferior suprarenal branches to the suprarenal
gland, the ureter, and the surrounding cellular tissue and muscles.

Which of the following regions of the male urethra is entirely


surrounded by Bucks fascia?
A Preprostatic part
.
B Prostatic part
.
C Membranous part
.
D Spongiose part
.
E. None of the above

Theme from 2010 Exam


Bucks fascia is a layer of deep fascia that covers the penis it is
continuous with the external spermatic fascia and the penile
suspensory ligament. The membranous part of the urethra may
partially pass through Bucks fascia as it passes into the penis.
However, the spongiose part of the urethra is contained wholly
within Bucks fascia.
Image of penile cross section
Bucks fascia corresponds to the layer of deep fascia

Image sourced from Wikipedia

Urethral anatomy
Female urethra
The female urethra is shorter and more acutely angulated than the
male urethra. It is an extra-peritoneal structure and embedded in
the endopelvic fascia. The neck of the bladder is subjected to
transmitted intra-abdominal pressure and therefore deficiency in
this area may result in stress urinary incontinence. Between the
layers of the urogenital diaphragm the female urethra is surrounded
by the external urethral sphincter, this is innervated by the
pudendal nerve. It ultimately lies anterior to the vaginal orifice.
Male urethra
In males the urethra is much longer and is divided into four parts.

Preprostatic
urethra

Extremely short and lies between the bladder and prostate


gland.It has a stellate lumen and is between 1 and 1.5cm
long.Innervated by sympathetic noradrenergic fibres, as this
region is composed of striated muscles bundles they may
contract and prevent retrograde ejaculation.

Prostatic
urethra

This segment is wider than the membranous urethra and


contains several openings for the transmission of semen (at
the midpoint of the urethral crest).

Membrano Narrowest part of the urethra and surrounded by external


us urethra sphincter. It traverses the perineal membrane 2.5cm posteroinferior to the symphysis pubis.
Penile
urethra

Travels through the corpus songiosum on the underside of


the penis. It is the longest urethral segment.It is dilated at its
origin as the infrabulbar fossa and again in the gland penis as
the navicular fossa. The bulbo-urethral glands open into the
spongiose section of the urethra 2.5cm below the perineal
membrane.

The urothelium is transitional in nature near to the bladder and


becomes squamous more distally.

48 year old lady is undergoing a left sided adrenalectomy for an adrenal


adenoma. The superior adrenal artery is injured and starts to bleed, from which
of the following does this vessel arise?
A. Left renal artery

B. Inferior phrenic artery


C. Aorta
D. Splenic
E. None of the above

The superior adrenal artery is a branch of the inferior phrenic artery.


Adrenal gland anatomy
Anatomy

Location
Relationships of the
right adrenal
Relationships of the
left adrenal
Arterial supply

Superomedially to the upper pole of each kidney


Diaphragm-Posteriorly, Kidney-Inferiorly, Vena CavaMedially, Hepato-renal pouch and bare area of the liverAnteriorly
Crus of the diaphragm-Postero- medially, Pancreas and
splenic vessels-Inferiorly, Lesser sac and stomach-Anteriorly
Superior adrenal arteries- from inferior phrenic artery,
Middle adrenal arteries - from aorta, Inferior adrenal arteries
-from renal arteries
Via one central vein directly into the IVC

Venous drainage of
the right adrenal
Venous drainage of
Via one central vein into the left renal vein
the left adrenal
Theme: Abdominal pain
A. Appendicitis
B. Threatened miscarriage
C. Ectopic pregnancy
D. Irritable bowel syndrome
E. Mittelschmerz
F. Pelvic inflammatory disease
G. Adnexial torsion
H. Endometriosis
I. Degenerating fibroid

Please select the most likely cause of abdominal pain for the clinical scenario given.
Each option may be used once, more than once or not at all.
5.

An 18 year-old girl presents to the Emergency Department with sudden onset


sharp, tearing pelvic pain associated with a small amount of vaginal bleeding.
She also complains of shoulder tip pain. On examination she is hypotensive,
tachycardic and has marked cervical excitation.

You answered Mittelschmerz


The correct answer is Ectopic pregnancy
The history of tearing pain and haemodynamic compromise in a women of
child bearing years should prompt a diagnosis of ectopic pregnancy.
6.

A 25 year-old lady presents to her GP complaining of a two day history of


right upper quadrant pain, fever and a white vaginal discharge. She has seen
the GP twice in 12 weeks complaining of pelvic pain and dyspareunia.
Pelvic inflammatory disease
The most likely diagnosis is pelvic inflammatory disease. Right upper
quadrant pain occurs as part of the Fitz Hugh Curtis syndrome in which peri
hepatic inflammation occurs.

7.

A 16 year old female presents to the emergency department with a 12 hour


history of pelvic discomfort. She is otherwise well and her last normal
menstrual period was 2 weeks ago. On examination she has a soft abdomen
with some mild supra pubic discomfort.
You answered Irritable bowel syndrome
The correct answer is Mittelschmerz
Mid cycle pain is very common and is due to the small amount of fluid
released during ovulation. Inflammatory markers are usually normal and the
pain typically subsides over the next 24-48 hours.

Gynaecological causes of abdominal pain


A number of women will present with abdominal pain and subsequently be diagnosed
with a gynaecological disorder. In addition to routine diagnostic work up of
abdominal pain, all female patients should also undergo a bimanual vaginal
examination, urine pregnancy test and consideration given to abdominal and pelvic
ultrasound scanning.
When diagnostic doubt persists a laparoscopy provides a reliable method of assessing
suspected tubulo-ovarian pathology.
Differential diagnoses of abdominal pain in females
Diagnosis
Features
Investigation
Treatment
Mittelschmerz Usually mid cycle
Full blood count- Conservative
pain.
usually normal

Often sharp onset.


Ultrasound- may
Little systemic
show small
disturbance.
quantity of free
May have recurrent
fluid
episodes.
Usually settles over
24-48 hours.
Endometriosis 25% asymptomatic, in Ultrasound- may
a further 25%
show free fluid
associated with other Laparoscopy will
pelvic organ
usually show
pathology.
lesions
Remaining 50% may
have menstrual
irregularity, infertility,
pain and deep
dyspareurina.
Complex disease may
result in pelvic
adhesional formation
with episodes of
intermittent small
bowel obstruction.
Intra-abdominal
bleeding may produce
localised peritoneal
inflammation.
Recurrent episodes are
common.
Ovarian
Usually sudden onset Ultrasound may
torsion
of deep seated colicky show free fluid
abdominal pain.
Laparoscopy is
Associated with
usually both
vomiting and distress. diagnostic and
Vaginal examination therapeutic
may reveal adnexial
tenderness.
Ectopic
Symptoms of
Ultrasound
gestation
pregnancy without
showing no intra
evidence of intra
uterine
uterine gestation.
pregnancy and
Present as an
beta HCG that is
emergency with
elevated
evidence of rupture or May show intra
impending rupture.
abdominal free
Open tubular ruptures fluid
may have sudden onset
of abdominal pain and
circulatory collapse, in
other the symptoms

Usually managed medically,


complex disease will often
require surgery and some
patients will even require
formal colonic and rectal
resections if these areas are
involved

Laparoscopy

Laparoscopy or laparotomy
is haemodynamically
unstable. A salphingectomy
is usually performed.

may be more
prolonged and less
marked.
Small amount of
vaginal discharge is
common.
There is usually
adnexial tenderness.
Pelvic
Bilateral lower
inflammatory abdominal pain
disease
associated with
vaginal discharge.
Dysuria may also be
present.
Peri-hepatic
inflammation
secondary to
Chlamydia (Fitz Hugh
Curtis Syndrome) may
produce right upper
quadrant discomfort.
Fever >38o

Full blood count- Usually medical


Leucocytosis
management
Pregnancy test
negative
(Although
infection and
pregnancy may
co-exist)
Amylase usually normal or
slightly raised
High vaginal and
urethral swabs

An 80 year old lady with a caecal carcinoma is undergoing a right


hemicolectomy performed through a transverse incision. The procedure is
difficult and the incision is extended medially by dividing the rectus
sheath. Brisk arterial haemorrhage ensues. From which of the following
does the damaged vessel originate?
A Internal iliac artery
.
B External iliac artery
.
C Superior vesical artery
.
D Inferior vesical artery
.
E. None of the above

The vessel damaged is the epigastric artery. This originates from the
external iliac artery (see below).

Epigastric artery
The inferior epigastric artery arises from the external iliac artery
immediately above the inguinal ligament. It then passes along the medial
margin of the deep inguinal ring. From here it continues superiorly to lie
behind the rectus abdominis muscle.
This is illustrated below:

Image sourced from Wikipedia

A 73 year old man has a large abdominal aortic aneurysm. During a


laparotomy for planned surgical repair the surgeons find the aneurysm is
far more proximally located and lies near the origin of the SMA. During the
dissection a vessel lying transversely across the aorta is injured. What is
this vessel most likely to be?
A Left renal vein
.
B Right renal vein
.

C Inferior mesenteric artery


.
D Ileocolic artery
.
E. Middle colic artery

Theme from April 2012 Exam


The left renal vein runs across the surface of the aorta and may require
deliberate ligation during juxtarenal aneurysm repair.
Abdominal aorta
Abdominal aortic topography
Origin

T12

Termination

L4

Posterior relations

L1-L4 Vertebral bodies

Anterior relations

Lesser omentum
Liver
Left renal vein
Inferior mesenteric vein
Third part of duodenum
Pancreas
Parietal peritoneum
Peritoneal cavity

Right lateral relations

Right crus of the diaphragm


Cisterna chyli
Azygos vein
IVC (becomes posterior distally)

Left lateral relations

4th part of duodenum


Duodenal-jejunal flexure
Left sympathetic trunk

The abdominal aorta

Image sourced from Wikipedia

A 18 year old man presents with an indirect inguinal hernia and undergoes
surgery. The deep inguinal ring is exposed and held with a retractor at its
medial aspect. Which structure is most likely to lie under the retractor?
A.

Ureter

B.

Inferior epigastric artery

C.

Internal iliac vein

D.

Femoral artery

E.

Lateral border of rectus abdominis

Boundaries of the deep inguinal ring:

Superolaterally - transversalis fascia

Inferomedially - inferior epigastric artery

The deep inguinal ring is closely related to the inferior epigastric artery.
The inferior epigastric artery forms part of the structure referred to as
Hesselbach's triangle.

Image sourced from Wikipedia

Inguinal canal
Location

Above the inguinal ligament

The inguinal canal is 4cm long

Boundaries of the inguinal canal


Floor
External oblique aponeurosis

Inguinal ligament

Lacunar ligament

Internal oblique

Transversus abdominis

Roof

Anterior wall

External oblique aponeurosis

Posterior wall

Transversalis fascia

Conjoint tendon

Internal ring

Fibres of internal oblique

External ring

Conjoint tendon

Laterally

Medially

Contents
Males Spermatic cord and
ilioinguinal nerve

As it passes through the canal the


spermatic cord has 3 coverings:

External spermatic fascia from


external oblique aponeurosis

Cremasteric fascia

Internal spermatic fascia

Femal Round ligament of uterus


es
and ilioinguinal nerve
Related anatomy of the inguinal region
The boundaries of Hesselbachs triangle are commonly tested and
illustrated below:

Image sourced from Wikipedia

The image below demonstrates the close relationship of the vessels to the
lower limb with the inguinal canal. A fact to be borne in mind when
repairing hernial defects in this region.

Image sourced from Wikipedia

1
2
Rate
question:

3
4
5

All contents of this site are 2012 E-Medical Revision Ltd


Conditions
Privacy policy

Terms and

In a patient with a carcinoma of the distal sigmoid colon, what is the most likely
source of its blood supply?
A. Ileocolic artery
B. External iliac artery
C. Internal iliac artery
D. Superior mesenteric artery
E. Inferior mesenteric artery
During a high anterior resection of such tumours, the inferior mesenteric artery is
ligated. Note that the branches (mainly middle rectal branch) of the internal iliac
artery are important in maintaining vascularity of the rectal stump and hence the
integrity of the anastomoses.
Rectum
The rectum is approximately 12 cm long. It is a capacitance organ. It has both intra
and extraperitoneal components. The transition between the sigmoid colon is marked
by the disappearance of the tenia coli.The extra peritoneal rectum is surrounded by
mesorectal fat that also contains lymph nodes. This mesorectal fatty layer is removed
surgically during rectal cancer surgery (Total Mesorectal Excision). The fascial layers
that surround the rectum are important clinical landmarks, anteriorly lies the fascia of
Denonvilliers. Posteriorly lies Waldeyers fascia.
Extra peritoneal rectum

Posterior upper third

Posterior and lateral middle third

Whole lower third

Relations
Anteriorly (Males)

Anteriorly (Females)
Posteriorly
Laterally

Rectovesical pouch
Bladder
Prostate
Seminal vesicles
Recto-uterine pouch (Douglas)
Cervix
Vaginal wall
Sacrum
Coccyx
Middle sacral artery
Levator ani
Coccygeus

Arterial supply
Superior rectal artery
Venous drainage
Superior rectal vein
Lymphatic drainage

Mesorectal lymph nodes (superior to dentate line)

Internal iliac and then para-aortic nodes

Inguinal nodes (inferior to dentate line)

A patient is due to undergo a right hemicolectomy for a carcinoma of the caecum.


Which of the following vessels will require high ligation to provide optimal
oncological control?
A. Middle colic artery
B. Inferior mesenteric artery
C. Superior mesenteric artery
D. Ileo-colic artery
E. None of the above
The ileo - colic artery supplies the caecum and would require high ligation during a
right hemicolectomy. The middle colic artery should generally be preserved when
resecting a caecal lesion.

This question is essentially asking you to name the vessel supplying the caecum. The
SMA does not directly supply the caecum, it is the ileocolic artery which does this.
Caecum
Location

Proximal right colon below the ileocaecal valve

Intraperitoneal

Psoas

Iliacus

Femoral nerve

Genitofemoral nerve

Gonadal vessels

Posterior relations

Anterior relations
Greater omentum
Arterial supply
Ileocolic artery
Lymphatic drainage Mesenteric nodes accompany the venous drainage

The caecum is the most distensible part of the colon and in complete large
bowel obstruction with a competent ileocaecal valve the most likely site of
eventual perforation.

A 72 year old man is undergoing a repair of an abdominal aortic aneurysm.


The aorta is cross clamped both proximally and distally. The proximal clamp
is applied immediately inferior to the renal arteries. Both common iliac
arteries are clamped distally. A longitudinal aortotomy is performed. After
evacuating the contents of the aneurysm sac a significant amount of ongoing
bleeding is encountered. This is most likely to originate from:
A. The coeliac axis
B. Testicular artery
C. Splenic artery
D. Superior mesenteric artery
E. Lumbar arteries

The lumbar arteries are posteriorly sited and are a common cause of back
bleeding during aortic surgery. The other vessels cited all exit the aorta in the
regions that have been cross clamped.
Abdominal aortic branches

Branches
Level
Paired
Type
Inferior phrenic
T12 (Upper border)
Yes
Parietal
Coeliac
T12
No
Visceral
Superior mesenteric
L1
No
Visceral
Middle suprarenal
L1
Yes
Visceral
Renal
L1-L2
Yes
Visceral
Gonadal
L2
Yes
Visceral
Lumbar
L1-L4
Yes
Parietal
Inferior mesenteric
L3
No
Visceral
Median sacral
L4
No
Parietal
Common iliac
L4
Yes
Terminal
The superficial inguinal ring is traversed by which of the following nerves?
A. Subcostal
B. Iliohypogastric
C. Ilioinguinal
D. Obturator
E. Pudendal
Ilioinguinal nerve entrapment may be a cause of neuropathic pain following inguinal
hernia surgery.
The ilioinguinal nerve passes through the superfical inguinal ring and is routinely
encountered when exploring the inguinal canal during hernia surgery. The
iliohypogastric nerve pierces the aponeurosis of the external oblique muscle superior
to the superficial inguinal ring.
Ilioinguinal nerve
Arises from the first lumbar ventral ramus with the iliohypogastric nerve. It passes
inferolaterally through the substance of psoas major and over the anterior surface of
quaratus lumborum. It pierces the internal oblique muscle and passes deep to the
aponeurosis of the external oblique muscle. It enters the inguinal canal and then
passes through the superficial inguinal ring to reach the skin.
Branches

To supply those muscles of the abdominal wall through which it passes.

Skin and fascia over the pubic symphysis, superomedial part of the femoral
triangle, surface of the scrotum, root and dorsum of penis or labum majus in
females.

A 63 year old man undergoes a radical cystectomy for carcinoma of the bladder.
During the procedure there is considerable venous bleeding. What is the primary site
of venous drainage of the urinary bladder?
A. Vesicoprostatic venous plexus
B. Internal iliac vein
C. External iliac vein
D. Gonadal vein
E. Common iliac vein
The urinary bladder has a rich venous plexus surrounding it, this drains subsequently
into the internal iliac vein. The vesicoprostatic plexus may be a site of considerable
venous bleeding during cystectomy.
Bladder
The empty bladder is contained within the pelvic cavity. It is usually a three sided
pyramid. The apex of the bladder points forwards towards the symphysis pubis and
the base lies immediately anterior to the rectum or vagina. Continuous with the apex
is the medial umbilical ligament, during development this was the site of the
uranchus.
The inferior aspect of the bladder is retroperitoneal and the superior aspect covered by
peritoneum. As the bladder distends it will tend to separate the peritoneum from the
fascia of tansversalis. For this reason a bladder that is distended due to acute urinary
retention may be approached with a suprapubic catheter that avoids entry into the
peritoneal cavity.
The trigone is the least mobile part of the bladder and forms the site of the ureteric
orifices and internal urethral orifice. In the empty bladder the ureteric orifices are
approximately 2-3cm apart, this distance may increase to 5cm in the distended
bladder.
Arterial supply
The superior and inferior vesical arteries provide the main blood supply to the
bladder. These are branches of the internal iliac artery.
Venous drainage
In males the bladder is drained by the vesicoprostatic venous plexus. In females the
bladder is drained by the vesicouterine venous plexus. In both sexes this venous
plexus will ultimately drain to the internal iliac veins.

Lymphatic drainage
Lymphatic drainage is predominantly to the external iliac nodes, internal iliac and
obturator nodes also form sites of bladder lymphatic drainage.
Innervation
Parasympathetic nerve fibres innervate the bladder from the pelvic splanchnic nerves.
Sympathetic nerve fibres are derived from L1 and L2 via the hypogastric nerve
plexuses. The parasympathetic nerve fibres will typically cause detrusor muscle
contraction and result in voiding. The muscle of the trigone is innervated by the
sympathetic nervous system. The external urethral sphincter is under concious
control. During bladder filling the rate of firing of nerve impulses to the detrusor
muscle is low and receptive relaxation occurs. At higher volumes and increased intra
vesical pressures the rate of neuronal firing will increase and eventually voiding will
occur.
A 60 year old female is undergoing a Whipples procedure for
adenocarcinoma of the pancreas. As the surgeons begin to mobilise the
pancreatic head they identify a large vessel passing inferiorly over the
anterior aspect of the pancreatic head. What is it likely to be?
A Superior mesenteric artery
.
B Coeliac axis
.
C Inferior mesenteric artery
.
D Aorta
.
E. Left gastric artery

Theme from January 2012 Exam


The superior mesenteric artery arises from the aorta and passes anterior
to the lower part of the pancreas. Invasion of this structure is a relative
contra indication to resectional surgery.
Pancreas
The pancreas is a retroperitoneal organ and lies posterior to the stomach.
It may be accessed surgically by dividing the peritoneal reflection that
connects the greater omentum to the transverse colon. The pancreatic
head sits in the curvature of the duodenum. It's tail lies close to the hilum
of the spleen, a site of potential injury during splenectomy.

Relations
Posterior to the pancreas
Pancreatic head

Inferior vena cava


Common bile duct
Right and left renal veins
Superior mesenteric vein and artery

Pancreatic neck

Superior mesenteric vein, portal vein

Pancreatic body-

Left renal vein


Crus of diaphragm
Psoas muscle
Adrenal gland
Kidney
Aorta

Pancreatic tail

Left kidney

Anterior to the pancreas


Pancreatic head

1st part of the duodenum


Pylorus
Gastroduodenal artery
SMA and SMV(uncinate process)

Pancreatic body

Stomach
Duodenojejunal flexure

Pancreatic tail

Splenic hilum

Superior to the pancreas


Coeliac trunk and its branches common hepatic artery and splenic artery
Grooves of the head of the pancreas
2nd and 3rd part of the duodenum
Arterial supply

Head: pancreaticoduodenal artery

Rest: splenic artery

Venous drainage

Head: superior mesenteric vein

Body and tail: splenic vein

Ampulla of Vater

Merge of pancreatic duct and common bile duct

Is an important landmark, halfway along the second part of the


duodenum, that marks the anatomical transition from foregut to
midgut (also the site of transition between regions supplied by
coeliac trunk and SMA).

Image sourced from Wikipedia

An 18 year old man is undergoing an orchidectomy via a scrotal approach. The


surgeons mobilise the spermatic cord. From which of the following is the outermost
layer of this structure derived?
A. Internal oblique aponeurosis
B. External oblique aponeurosis
C. Transversalis fascia
D. Rectus sheath
E. Campers fascia
The outermost covering of the spermatic cord is derived from the external oblique
aponeurosis.This layer is added as the cord passes through the superficial inguinal
ring.

Scrotal and testicular anatomy


Spermatic cord
Formed by the vas deferens and is covered by the following structures:
Layer
Origin
Internal spermatic fascia
Transversalis fascia
Cremasteric fascia
From the fascial coverings of internal oblique
External spermatic fascia
External oblique aponeurosis
Contents of the cord
Vas deferens
Testicular artery
Artery of vas deferens
Cremasteric artery
Pampiniform plexus
Sympathetic nerve fibres
Genital branch of the
genitofemoral nerve
Lymphatic vessels

Transmits sperm and accessory gland


secretions
Branch of abdominal aorta supplies testis and
epididymis
Arises from inferior vesical artery
Arises from inferior epigastic artery
Venous plexus, drains into right or left
testicular vein
Lie on arteries, the parasympathetic fibres lie
on the vas
Supplies cremaster
Drain to lumbar and para-aortic nodes

Scrotum

Composed of skin and closely attached dartos fascia.

Arterial supply from the anterior and posterior scrotal arteries

Lymphatic drainage to the inguinal lymph nodes

Parietal layer of the tunica vaginalis is the innermost layer

Testes

The testes are surrounded by the tunica vaginalis (closed peritoneal sac). The
parietal layer of the tunica vaginalis adjacent to the internal spermatic fascia.

The testicular arteries arise from the aorta immediately inferiorly to the renal
arteries.

The pampiniform plexus drains into the testicular veins, the left drains into the
left renal vein and the right into the inferior vena cava.

Lymphatic drainage is to the para-aortic nodes.

A 53 year old male presents with a carcinoma of the transverse colon. Which of the
following structures should be ligated close to their origin to maximise clearance of
the tumour?
A. Superior mesenteric artery
B. Inferior mesenteric artery
C. Middle colic artery
D. Ileo-colic artery
E. Superior rectal artery
The middle colic artery supplies the transverse colon and requires high ligation during
cancer resections. It is a branch of the superior mesenteric artery.
Transverse colon

The right colon undergoes a sharp turn at the level of the hepatic flexure to
become the transverse colon.

At this point it also becomes intraperitoneal.

It is connected to the inferior border of the pancreas by the transverse


mesocolon.

The greater omentum is attached to the superior aspect of the transverse colon
from which it can easily be separated. The mesentery contains the middle colic
artery and vein. The greater omentum remains attached to the transverse colon
up to the splenic flexure. At this point the colon undergoes another sharp turn.

Relations
Superior Liver and gall-bladder, the greater curvature of the stomach, and the lower
end of the spleen
Inferior Small intestine
Anterior Greater omentum
Posterior From right to left with the descending portion of the duodenum, the head of
the pancreas, convolutions of the jejunum and ileum, spleen
Which of the following structures does not lie posterior to the right kidney?
A. Psoas major
B. Transversus abdominis
C. Quadratus lumborum

D. Medial artcuate ligament


E. 10th rib
Theme from April 2012 Exam
The 10th rib lies more superior. The 12th rib is a closer relation posteriorly.
Renal anatomy
Each kidney is about 11cm long, 5cm wide and 3cm thick. They are located in a deep
gutter alongside the projecting verterbral bodies, on the anterior surface of psoas
major. In most cases the left kidney lies approximately 1.5cm higher than the right.
The upper pole of both kidneys approximates with the 11th rib (beware pneumothorax
during nephrectomy). On the left hand side the hilum is located at the L1 vertebral
level and the right kidney at level L1-2. The lower border of the kidneys is usually
alongside L3.
The table below shows the anatomical relations of the kidneys:
Relations
Relations Right Kidney
Posterior Quadratus lumborum, diaphragm,
psoas major, transversus abdominis
Anterior Hepatic flexure of colon
Superior Liver, adrenal gland

Left Kidney
Quadratus lumborum, diaphragm,
psoas major, transversus abdominis
Stomach, Pancreatic tail
Spleen, adrenal gland

Fascial covering
Each kidney and suprarenal gland is enclosed within a common and layer of investing
fascia that is derived from the transversalis fascia into anterior and posterior layers
(Gerotas fascia).
Renal structure
Kidneys are surrounded by an outer cortex and an inner medulla which usually
contains between 6 and 10 pyramidal structures. The papilla marks the innermost apex
of these. They terminate at the renal pelvis, into the ureter.
Lying in a hollow within the kidney is the renal sinus. This contains:
1. Branches of the renal artery
2. Tributaries of the renal vein
3. Major and minor calyces's
4. Fat
Structures at the renal hilum
The renal vein lies most anteriorly, then renal artery (it is an end artery) and the ureter
lies most posterior.
A 73 year old lady presents with a femoral hernia. Which of the following
structures forms the lateral wall of the femoral canal?

A Pubic tubercle
.
B Femoral vein
.
C Femoral artery
.
D Conjoint tendon
.
E. Femoral nerve

The canal exists to allow for the physiological expansion of the femoral
vein, which lies lateral to it.
Femoral canal
The femoral canal lies at the medial aspect of the femoral sheath. The
femoral sheath is a fascial tunnel containing both the femoral artery
laterally and femoral vein medially. The canal lies medial to the vein.
Borders of the femoral canal
Laterally

Femoral vein

Medially

Lacunar ligament

Anteriorly

Inguinal ligament

Posteriorly

Pectineal ligament

Image showing dissection of femoral canal

Image sourced from Wikipedia

Contents

Lymphatic vessels

Cloquet's lymph node

Physiological significance
Allows the femoral vein to expand to allow for increased venous return to
the lower limbs.
Pathological significance
As a potential space, it is the site of femoral hernias. The relatively tight
neck places these at high risk of strangulation.

How many unpaired branches leave the abdominal aorta to supply the abdominal
viscera?
A. One

B. Two
C. Three
D. Four
E. Five
There are three unpaired branches to the abdominal viscera. These include the coeliac
axis, the SMA and IMA. Branches to the adrenals, renal arteries and gonadal vessels
are paired. The fourth unpaired branch of the abdominal aorta, the median sacral
artery, does not directly supply the abdominal viscera.
Abdominal aortic branches
Branches
Inferior phrenic
Coeliac
Superior mesenteric
Middle suprarenal
Renal
Gonadal
Lumbar
Inferior mesenteric
Median sacral
Common iliac

Level
T12 (Upper border)
T12
L1
L1
L1-L2
L2
L1-L4
L3
L4
L4

Paired
Yes
No
No
Yes
Yes
Yes
Yes
No
No
Yes

Type
Parietal
Visceral
Visceral
Visceral
Visceral
Visceral
Parietal
Visceral
Parietal
Terminal

A 25 year old man is being catheterised, prior to a surgical procedure. As


the catheter enters the prostatic urethra which of the following changes
will occur?
A Resistance will increase significantly
.
B Resistance will increase slightly
.
C It will lie horizontally
.
D Resistance will decrease
.
E. It will deviate laterally

Theme from September 2011 Exam


The prostatic urethra is much wider than the membranous urethra and
therefore resistance will decrease. The prostatic urethra is inclined
vertically.
Prostate gland
The prostate gland is approximately the shape and size of a walnut and is
located inferior to the bladder. It is separated from the rectum by
Denonvilliers fascia and its blood supply is derived from the internal iliac
vessels. The internal sphincter lies at the apex of the gland and may be
damaged during prostatic surgery, affected individuals may complain of
retrograde ejaculation.
Summary of prostate gland
Arterial
supply

Inferior vesical artery (from internal iliac)

Venous
drainage

Prostatic venous plexus (to paravertebral veins)

Lymphatic
drainage

Internal iliac nodes

Innervation Inferior hypogastric plexus


Dimensions

Transverse diameter (4cm)

AP diameter (2cm)

Height (3cm)

Posterior lobe: posterior to urethra

Median lobe: posterior to urethra, in between


ejaculatory ducts

Lateral lobes x 2

Isthmus

Peripheral zone: subcapsular portion of posterior


prostate. Most prostate cancers are here

Lobes

Zones

Central zone

Transition zone

Stroma

Relations
Anterior Pubic symphysis
Prostatic venous plexus
Posterior Denonvilliers fascia
Rectum
Ejaculatory ducts
Lateral

Venous plexus (lies on prostate)


Levator ani (immediately below the puboprostatic ligaments)

Image sourced from Wikipedia

A 78 year old man develops a carcinoma of the scrotum. To which of the following
lymph node groups may the tumour initially metastasise?
A. Para aortic
B. Obturator
C. Inguinal

D. Meso rectal
E. None of the above
The scrotum is drained by the inguinal nodes.
Scrotal and testicular anatomy
Spermatic cord
Formed by the vas deferens and is covered by the following structures:
Layer
Origin
Internal spermatic fascia
Transversalis fascia
Cremasteric fascia
From the fascial coverings of internal oblique
External spermatic fascia
External oblique aponeurosis
Contents of the cord
Vas deferens
Testicular artery
Artery of vas deferens
Cremasteric artery
Pampiniform plexus
Sympathetic nerve fibres
Genital branch of the
genitofemoral nerve
Lymphatic vessels

Transmits sperm and accessory gland


secretions
Branch of abdominal aorta supplies testis and
epididymis
Arises from inferior vesical artery
Arises from inferior epigastic artery
Venous plexus, drains into right or left
testicular vein
Lie on arteries, the parasympathetic fibres lie
on the vas
Supplies cremaster
Drain to lumbar and para-aortic nodes

Scrotum

Composed of skin and closely attached dartos fascia.

Arterial supply from the anterior and posterior scrotal arteries

Lymphatic drainage to the inguinal lymph nodes

Parietal layer of the tunica vaginalis is the innermost layer

Testes

The testes are surrounded by the tunica vaginalis (closed peritoneal sac). The
parietal layer of the tunica vaginalis adjacent to the internal spermatic fascia.

The testicular arteries arise from the aorta immediately inferiorly to the renal
arteries.

The pampiniform plexus drains into the testicular veins, the left drains into the
left renal vein and the right into the inferior vena cava.

Lymphatic drainage is to the para-aortic nodes.

An 22 year old soldier is shot in the abdomen and amongst his various injuries
is a major disruption to the abdominal aorta. There is torrential haemorrhage
and the surgeons decide to control the aorta by placement of a vascular clamp
immediately inferior to the diaphragm. Which of the following vessels may be
injured in this maneouvre?
A. Inferior phrenic arteries
B. Superior phrenic arteries
C. Splenic artery
D. Renal arteries
E. Superior mesenteric artery

As the first branches of the abdominal aorta the inferior phrenic arteries are at
greatest risk. The superior phrenic arteries lie in the thorax. The potential
space at the level of the diaphragmatic hiatus is a potentially useful site for
aortic occlusion. However, leaving the clamp applied for more than about 10
-15 minutes usually leads to poor outcomes.
Abdominal aorta

Abdominal aortic topography


Origin
T12
Termination
L4
Posterior relations
L1-L4 Vertebral bodies
Anterior relations
Lesser omentum
Liver
Left renal vein
Inferior mesenteric vein
Third part of duodenum
Pancreas
Parietal peritoneum
Peritoneal cavity
Right lateral relations
Right crus of the diaphragm
Cisterna chyli
Azygos vein
IVC (becomes posterior distally)
Left lateral relations
4th part of duodenum

Duodenal-jejunal flexure
Left sympathetic trunk

The abdominal aorta

Image sou

Which of the following statements relating to the gallbladder is untrue?


A. The fundus is usually intra peritoneal
B. Arterial supply is from the cystic artery
C. The cystic artery is usually located in Calots triangle
D. Calots triangle may rarely contain an aberrant hepatic artery
E. Cholecystokinin causes relaxation of the gallbladder
CCK causes gallbladder contraction.
Gallbladder

Fibromuscular sac with capacity of 50ml

Columnar epithelium

Relations of the gallbladder

Anterior
Posterior

Laterally
Medially

Liver

Covered by peritoneum

Transverse colon

1st part of the duodenum

Right lobe of liver


Quadrate lobe of liver

Arterial supply
Cystic artery (branch of Right hepatic artery)
Venous drainage
Cystic vein
Nerve supply
Sympathetic- mid thoracic spinal cord, Parasympathetic- anterior vagal trunk
Common bile duct
Origin
Relations at
origin

Relations
distally

Confluence of cystic and common hepatic ducts

Medially - Hepatic artery

Posteriorly- Portal vein

Duodenum - anteriorly

Pancreas - medially and laterally

Right renal vein posteriorly

Arterial supply Branches of hepatic artery and retroduodenal branches of


gastroduodenal artery
Calot's triangle
Medially
Inferiorly
Superiorly
Contents

Common hepatic duct


Cystic duct
Inferior edge of liver
Cystic artery

Which of the following is not contained within the deep posterior


compartment of the lower leg?

A.

Tibialis posterior muscle

B.

Posterior tibial artery

C.

Tibial nerve

D.

Sural nerve

E.

Flexor hallucis longus

The deep posterior compartment lies anterior to soleus. The sural nerve is
superficially sited and therefore not contained within it.

Lower limb- Muscular compartments

Anterior compartment
Muscle

Nerve Action

Tibialis anterior
inverts foot

Deep peroneal nerve

Dorsiflexes ankle joint,

Extensor digitorum longus


Deep peroneal nerve
four toes, dorsiflexes ankle joint
Peroneus tertius

Deep peroneal nerve

Extends lateral

Dorsiflexes ankle, everts foot

Extensor hallucis longus Deep peroneal nerve


extends big toe

Dorsiflexes ankle joint,

Peroneal compartment
Muscle

Nerve Action

Peroneus longus
plantar flexion

Superficial peroneal nerve

Everts foot, assists in

Peroneus brevis
ankle joint

Superficial peroneal nerve

Plantar flexes the

Superficial posterior compartment


Nerve Action

Gastrocnemius
knee
Soleus

Tibial nerve Plantar flexes the foot, may also flex the

Tibial nerve Plantar flexor

Deep posterior compartment


Muscle

Nerve Action

Flexor digitorum longus

Tibial Flexes the lateral four toes

Flexor hallucis longus

Tibial Flexes the great toe

Tibialis posterior

Tibial Plantar flexor, inverts the foot

A 25 year old man is being catheterised, prior to a surgical procedure. As


the catheter enters the prostatic urethra which of the following changes
will occur?

A.

Resistance will increase significantly

B.

Resistance will increase slightly

C.

It will lie horizontally

D.

Resistance will decrease

E.

It will deviate laterally

Theme from September 2011 Exam

The prostatic urethra is much wider than the membranous urethra and
therefore resistance will decrease. The prostatic urethra is inclined
vertically.

Prostate gland

The prostate gland is approximately the shape and size of a walnut and is
located inferior to the bladder. It is separated from the rectum by

Denonvilliers fascia and its blood supply is derived from the internal iliac
vessels. The internal sphincter lies at the apex of the gland and may be
damaged during prostatic surgery, affected individuals may complain of
retrograde ejaculation.

Summary of prostate gland


Arterial supply

Inferior vesical artery (from internal iliac)

Venous drainage

Prostatic venous plexus (to paravertebral veins)

Lymphatic drainage

Internal iliac nodes

Innervation Inferior hypogastric plexus


Dimensions

Transverse diameter (4cm)


AP diameter (2cm)
Height (3cm)

Lobes

Posterior lobe: posterior to urethra


Median lobe: posterior to urethra, in between ejaculatory ducts
Lateral lobes x 2
Isthmus

Zones

Peripheral zone: subcapsular portion of posterior prostate. Most prostate


cancers are here
Central zone
Transition zone
Stroma

Relations
Anterior

Pubic symphysis

Prostatic venous plexus


Posterior

Denonvilliers fascia

Rectum
Ejaculatory ducts
Lateral

Venous plexus (lies on prostate)

Levator ani (immediately below the puboprostatic ligaments)

[http://cdn.emrcs.com/images_eMRCS/swb129b.png]
Image sourced from Wikipedia
[http://cdn.emrcs.com/images_eMRCS/swb129b.png]

Which of the following nerves is the primary source of innervation to the anterior
scrotal skin?
A. Genital branch of the genitofemoral nerve
B. Pudendal nerve
C. Ilioinguinal nerve
D. Femoral branch of the genitofemoral nerve
E. Obturator nerve
Theme from April 2012 Exam
The pudendal nerve may innervate the posterior skin of the scrotum. The anterior
innervation of the scrotum is primarily provided by the ilioinguinal nerve. The genital
branch of the genitofemoral nerve provides a smaller contribution.
Scrotal sensation
The scrotum is innervated by the ilioinguinal nerve and the pudendal nerve. The
ilioinguinal nerve arises from L1 and pierces the internal oblique muscle. It eventually
passes through the superficial inguinal ring to innervate the anterior skin of the
scrotum.

The pudendal nerve is the principal nerve of the perineum. It arises in the pelvis from
3 nerve roots. It passes through both greater and lesser sciatic foramina to enter the
perineal region. The perineal branches pass anteromedially and divide into posterior
scrotal branches. The posterior scrotal branches pass superficially to supply the skin
and fascia of the perineum. It cross communicates with the inferior rectal nerve.
transversalis fascia contributes to which of the following?
A Pectineal ligament
.
B Deep inguinal ring
.
C Cremaster muscle and fascia
.
D Inguinal ligament
.
E External spermatic fascia
.
The internal spermatic fascia (derived from transversalis fascia) invests:
Ducuts deferens
Testicular vessels
The principal outpouching of the transversalis fascia is the internal
spermatic fascia. The mouth of the outpouching is the deep inguinal ring.
Abdominal wall
The 2 main muscles of the abdominal wall are the rectus abdominis
(anterior) and the quadratus lumborum (posterior).
The remaining abdominal wall consists of 3 muscular layers. Each muscle
passes from the lateral aspect of the quadratus lumborum posteriorly to
the lateral margin of the rectus sheath anteriorly. Each layer is muscular
posterolaterally and aponeurotic anteriorly.

Image sourced from Wikipedia

Muscles of abdominal wall


External
Lies most superficially
oblique

Internal
oblique

Transversu
s
abdominis

Originates from 5th to 12th ribs

Inserts into the anterior half of the outer aspect of the


iliac crest, linea alba and pubic tubercle

More medially and superiorly to the arcuate line, the


aponeurotic layer overlaps the rectus abdominis
muscle

The lower border forms the inguinal ligament

The triangular expansion of the medial end of the


inguinal ligament is the lacunar ligament.

Arises from the thoracolumbar fascia, the anterior 2/3


of the iliac crest and the lateral 2/3 of the inguinal
ligament

The muscle sweeps upwards to insert into the


cartilages of the lower 3 ribs

The lower fibres form an aponeurosis that runs from


the tenth costal cartilage to the body of the pubis

At its lowermost aspect it joins the fibres of the


aponeurosis of transversus abdominis to form the
conjoint tendon.

Innermost muscle

Arises from the inner aspect of the costal cartilages of


the lower 6 ribs , from the anterior 2/3 of the iliac crest
and lateral 1/3 of the inguinal ligament

Its fibres run horizontally around the abdominal wall


ending in an aponeurosis. The upper part runs
posterior to the rectus abdominis. Lower down the
fibres run anteriorly only.

The rectus abdominis lies medially running from the


pubic crest and symphysis to insert into the xiphoid
process and 5th, 6th and 7th costal cartilages. The
muscles lies in a aponeurosis as described above.

Nerve supply: anterior primary rami of T7-12

Surgical notes
During abdominal surgery it is usually necessary to divide either the
muscles or their aponeuroses. During a midline laparotomy it is desirable
to divide the aponeurosis. This will leave the rectus sheath intact above
the arcuate line and the muscles intact below it. Straying off the midline
will often lead to damage to the rectus muscles, particularly below the
arcuate line where they may often be in close proximity to each other.

A 73 year old lady is admitted with right iliac fossa pain. A plain abdominal x-ray is
taken and the caecal diameter measured. Which of the following caecal diameters are
pathological?
A. 4cm
B. 5cm
C. 6cm
D. 7cm
E. 10cm
8 cm is still within normal limits. However, caecal diameters of 9 and 10 are
pathological and should prompt further investigation.
Right colon
Ileocaecal valve

Entry point of the terminal ileum to the caecum

An important colonoscopic landmark

The ileocaecal valve is not always competent and this may allow partial
decompression of an obstructed colon

Appendix

At the base of the caecum the taenia coalesce to mark the base of the appendix

This is a reliable way of locating the appendix surgically and is a constant


landmark

The appendix has a small mesentery (the mesoappendix) and in this runs the
appendiceal artery, a branch of the ileocolic artery.

The posterior aspect of the right colon is extra peritoneal and the anterior aspect
intraperitoneal.
Relations

Posterior

Iliacus, Iliolumbar ligament, Quadratus lumborum, Transverse abdominis, Diaphragm


at the tip of the last rib; ` Lateral cutaneous, ilioinguinal, and iliohypogastric nerves;
the iliac branches of the iliolumbar vessels, the fourth lumbar artery, gonadal vessels,
ureter and the right kidney.

Superior

Right kidney which is embedded in the perinephric fat

Medial

Mesentery which contains the ileocolic artery that supplies the right colon and
terminal ileum. A further branch , the right colic artery, also contributes to supply the
hepatic flexure and proximal transverse colon. Medially these pass through the
mesentery to join the SMA. This occurs near to the head of the pancreas and care has
to be taken when ligating the ileocolic artery near to its origin in cancer cases for fear
of impinging on the SMA.
- Anterior
Coils of small intestine, the right edge of the greater omentum, and the anterior
abdominal wall.
Nerve supply

Parasympathetic fibres of the vagus nerve (CN X)

Arterial supply

Ileocolic artery and right colic artery, both branches of the SMA. While the
ileocolic artery is almost always present, the right colic can be absent in 515% of individuals.

Which of the following options in relation to the liver is true?


A. Ligamentum venosum is an anterior relation
of the liver
B. The portal triad comprises the hepatic artery,
hepatic vein and tributary of the bile duct
C. The liver is completely covered by
peritoneum
D. There are no nerves within the porta hepatis
E. The caudate lobe is superior to the porta
hepatis
'VC goes with VC'
The ligamentun Venosum and Caudate is on same side as Vena Cava [posterior].
Ligamentum venosum is posterior to the liver. The portal triad contains the portal vein
rather than the hepatic vein. There is the 'bare area of the liver' created by a void due
to the coronary ligament layers being widely separated. There are sympathetic and
parasympathetic nerves in the porta hepatis.
Liver
Structure of the liver
Right lobe
Supplied by right hepatic artery

Contains Couinard segments V to VIII (-/+Sg I)

Supplied by the left hepatic artery

Contains Couinard segments II to IV (+/- Sg1)

Part of the right lobe anatomically, functionally is part of the


left

Left lobe

Quadrate
lobe

Couinard segment IV

Porta hepatis lies behind

On the right lies the gallbladder fossa

On the left lies the fossa for the umbilical vein

Supplied by both right and left hepatic arteries

Couinard segment I

Lies behind the plane of the porta hepatis

Anterior and lateral to the inferior vena cava

Bile from the caudate lobe drains into both right and left
hepatic ducts

Caudate lobe

Detailed knowledge of Couinard segments is not required for MRCS Part A

Between the liver lobules are portal canals which contain the portal triad:
Hepatic Artery, Portal Vein, tributary of Bile Duct.

Relations of the liver


Anterior
Diaphragm
Xiphoid process

Postero inferiorly
Oesophagus
Stomach
Duodenum
Hepatic flexure of colon
Right kidney
Gallbladder
Inferior vena cava

Porta hepatis
Location Postero inferior surface, it joins nearly at right angles with the left sagittal
fossa, and separates the caudate lobe behind from the quadrate lobe in
front
Transmits
Common hepatic duct

Hepatic artery

Portal vein

Sympathetic and parasympathetic nerve fibres

Lymphatic drainage of the liver (and nodes)

Ligaments
Falciform ligament

2 layer fold peritoneum from the umbilicus to anterior


liver surface

Contains ligamentum teres (remnant umbilical vein)

On superior liver surface it splits into the coronary and


left triangular ligaments

Ligamentum teres Joins the left branch of the portal vein in the porta hepatis
Ligamentum
Remnant of ductus venosus
venosum
Arterial supply

Hepatic artery

Venous

Hepatic veins

Portal vein

Nervous supply

Sympathetic and parasympathetic trunks of coeliac plexus

he following statements regarding the rectus abdominis muscle are true


except:
A It runs from the symphysis pubis to the xiphoid process
.

B Its nerve supply is from the ventral rami of the lower 6 thoracic
. nerves
C It has collateral supply from both superior and inferior
. epigastric vessels
D It lies in a muscular aponeurosis throughout its length
.
E. It has a number of tendinous intersections that penetrate
through the anterior layer of the muscle
The aponeurosis is deficient below the arcuate line.
Rectus abdominis:
Arises from 5th, 6th, 7th costal cartilages.
Inserts into the pubis.
The muscle lies in the rectal sheath, which also contains the superior &
inferior epigastric artery and vein.
Action: flexion of thoracic and lumbar spine.
Nerve supply: anterior primary rami of T7-12
Abdominal wall
The 2 main muscles of the abdominal wall are the rectus abdominis
(anterior) and the quadratus lumborum (posterior).
The remaining abdominal wall consists of 3 muscular layers. Each muscle
passes from the lateral aspect of the quadratus lumborum posteriorly to
the lateral margin of the rectus sheath anteriorly. Each layer is muscular
posterolaterally and aponeurotic anteriorly.

Image sourced from Wikipedia

Muscles of abdominal wall


External
Lies most superficially
oblique

Originates from 5th to 12th ribs

Internal
oblique

Transversu
s
abdominis

Inserts into the anterior half of the outer aspect of the


iliac crest, linea alba and pubic tubercle

More medially and superiorly to the arcuate line, the


aponeurotic layer overlaps the rectus abdominis
muscle

The lower border forms the inguinal ligament

The triangular expansion of the medial end of the


inguinal ligament is the lacunar ligament.

Arises from the thoracolumbar fascia, the anterior 2/3


of the iliac crest and the lateral 2/3 of the inguinal
ligament

The muscle sweeps upwards to insert into the


cartilages of the lower 3 ribs

The lower fibres form an aponeurosis that runs from


the tenth costal cartilage to the body of the pubis

At its lowermost aspect it joins the fibres of the


aponeurosis of transversus abdominis to form the
conjoint tendon.

Innermost muscle

Arises from the inner aspect of the costal cartilages of


the lower 6 ribs , from the anterior 2/3 of the iliac crest
and lateral 1/3 of the inguinal ligament

Its fibres run horizontally around the abdominal wall


ending in an aponeurosis. The upper part runs
posterior to the rectus abdominis. Lower down the
fibres run anteriorly only.

The rectus abdominis lies medially running from the


pubic crest and symphysis to insert into the xiphoid
process and 5th, 6th and 7th costal cartilages. The
muscles lies in a aponeurosis as described above.

Nerve supply: anterior primary rami of T7-12

Surgical notes
During abdominal surgery it is usually necessary to divide either the
muscles or their aponeuroses. During a midline laparotomy it is desirable

to divide the aponeurosis. This will leave the rectus sheath intact above
the arcuate line and the muscles intact below it. Straying off the midline
will often lead to damage to the rectus muscles, particularly below the
arcuate line where they may often be in close proximity to each other.
A 42 year old male sustains a back injury resulting in the compression of
the conus medullaris. Which of the dematomes below is most likely to be
affected by this process?
A S1
.
B L1
.
C S3
.
D L3
.
E. L5

Theme from the September 2012 Exam


The perineum is innervated by S3 and S4, S2 runs down the posterior
aspect of the leg and would also be affected.
Dermatomes
The common dermatomal levels and cutaneous nerves responsible for
them is illustrated below.

Image sourced from Wikipedia

During liver mobilisation for a cadaveric liver transplant the hepatic ligaments will
require mobilisation. Which of the following statements relating to these structures is
untrue?
A. Lesser omentum arises from the porta hepatis and passes the lesser
curvature of the stomach
B. The falciform ligament divides into the left triangular ligament and
coronary ligament
C. The liver has an area devoid of peritoneum

D. The coronary ligament is attached to the liver


E. The right triangular ligament is an early branch of the left triangular
ligament
The right triangular ligament is a continuation of the coronary ligament.
Liver
Structure of the liver
Right lobe
Supplied by right hepatic artery

Contains Couinard segments V to VIII (-/+Sg I)

Supplied by the left hepatic artery

Contains Couinard segments II to IV (+/- Sg1)

Part of the right lobe anatomically, functionally is part of the


left

Couinard segment IV

Porta hepatis lies behind

On the right lies the gallbladder fossa

On the left lies the fossa for the umbilical vein

Supplied by both right and left hepatic arteries

Couinard segment I

Lies behind the plane of the porta hepatis

Anterior and lateral to the inferior vena cava

Bile from the caudate lobe drains into both right and left
hepatic ducts

Left lobe

Quadrate
lobe

Caudate lobe

Detailed knowledge of Couinard segments is not required for MRCS Part A

Between the liver lobules are portal canals which contain the portal triad:
Hepatic Artery, Portal Vein, tributary of Bile Duct.

Relations of the liver


Anterior
Diaphragm
Xiphoid process

Postero inferiorly
Oesophagus
Stomach
Duodenum
Hepatic flexure of colon
Right kidney
Gallbladder
Inferior vena cava

Porta hepatis
Location Postero inferior surface, it joins nearly at right angles with the left sagittal
fossa, and separates the caudate lobe behind from the quadrate lobe in
front
Transmits
Common hepatic duct

Hepatic artery

Portal vein

Sympathetic and parasympathetic nerve fibres

Lymphatic drainage of the liver (and nodes)

Ligaments
Falciform ligament

2 layer fold peritoneum from the umbilicus to anterior


liver surface

Contains ligamentum teres (remnant umbilical vein)

On superior liver surface it splits into the coronary and


left triangular ligaments

Ligamentum teres Joins the left branch of the portal vein in the porta hepatis
Ligamentum
Remnant of ductus venosus
venosum
Arterial supply

Hepatic artery

Venous

Hepatic veins

Portal vein

Nervous supply

Sympathetic and parasympathetic trunks of coeliac plexus

Anda mungkin juga menyukai